Sie sind auf Seite 1von 79

BUSINESS ASSOCIATIONS OUTLINE WELLS [ELH] Grade - 94

Agency
o Definition A fiduciary relationship that arises when one person manifests assent to another person that the agent shall act on the principals behalf and subject to the principals control and the agent manifests assent or otherwise consents so to act Requires two people Principal o Hiring entity o Disclosed Third party knows that the agent is acting for a principal and who that principal is o Undisclosed Third party doesnt know that the agent is acting for a principal o Unidentified Third party knows that the agent is acting for a principal but doesnt know who the principal is Agent o Hired person o Co-agents Share the same principal o Dual Agent Acts on behalf of more than one principal (joint principals) for the same transaction Fiduciary relationship Fiduciary faithful, look out for other persons interests rather than your own o If it comes down to something that is good for me and something that is good for you I am supposed to do whats good for you Limits o Illegal actions o General harmful actions One person asks another person to act for her and the other person consents to act Consent can be manifested through actions or words The principal has some control over the agent Actual Authority Agent reasonably believes that the principal wishes the agent to act as he is Interpretation is reasonable if: o Reflects a meaning that principal would give o In the absence of a known meaning, if a reasonable person in agents position would interpret the manifestations in light of the context Scope Designated or implied by principals manifestations and acts necessary or incidental to achieving principals objectives at the time the action was taken Apparent Authority 1

Power held by agent to affect principals legal relations with third parties when third party reasonably believes actor has authority to act on behalf of principal and that belief is traceable to the principals manifestation Straight forward regular old agents Employer-employee relationship Employer has a great deal of control over basically every aspect fo the conditions of the employment Employees are fiduciaries of the employers Independent Contractor Lawyer-client relationship Client has a certain degree of control over the lawyer Client is the principal, lawyer is the agent Client is not liable for the lawyers behavior as a normal principal would be Other independent contractors Doctors, plumbers, repair people Engage them for a certain task, contractual relationship, some control over what they do, but the contractor has a lot of control over how they do it They are engaged in their own business, not within your business Issues Fiduciary relationship Partners are fiduciaries to one another Employees are fiduciaries to partnerships and corporations Officers and directors owe fiduciary duties to their corporations Is the employer (principal) responsible for what the agent does? Respondeat superior o Principal is liable for torts committed by agent while acting in the scope of their employment o Agent is also responsible for their own negligence o Focus on the principal because the principal usually has the money What if the agent violated company rules when they committed the tort? o Employer is still responsible o They are getting a social good if youre going to use other people in your business then you have to be responsible for what they do in the course of your use of them o Business may be in a better position to insure against this can pass that cost along to their customers through the price of their goods o If the employee commits a crime then the employer may not be responsible Contract issues Employee does something that injures the employer An employee who is going to sign a contract on behalf of the company may have to get an ok from the board of directors What if the employee does not get the ok? o Authority is the issue need to know if the employee has the authority to sign the contract on their own Estoppel to deny existence of agency relationship Person that has not manifested that an actor has agency power is liable to third party who is justifiably induced to make a detrimental change in position b/c transaction is believed to be on persons account if: o Person intentionally or carelessly caused such belief OR o Having notice and that it might induce others to change their positions, the person did not take reasonable steps to notify them of the facts Estoppel of undisclosed principal 2

Undisclosed principal cannot reduce agents authority to less than a third party would reasonably believe the agent to have under the same circumstances had the principal been disclosed

Sole Proprietorship
o o o o o o Simplest form of business organization One owner has ultimate control and veto power Personally liable for all business obligations No legal separation between owner and business Agency, employment, tort and contract law govern Uniform Partnership Act Applicable UPA Provisions 2 6 definition 7 rules for determining the existence of a partnership 9 13 14 16 17 18 rules determining rights and duties of partners 20 21 22 25 26 29 30 31 32 35(1)(b) 36 38 40 Rules for Distribution upon Dissolution 41 42 Applicable RUPA Provisions 306 1001 1002 1003 Whatever is not included in the partnership agreement is completed by the statute Gap fillers Statute has some mandatory provisions and some amendable provisions Overview Decentralized management o Everybody thats an owner gets to participate Pass-through (conduit) taxation 3

Partnership

o General Partnership

Unlimited personal liability Limited life o Cannot last forever o Partnerships are fragile death, bankruptcy, departure of a partner could all end a partnership Broad agent authority o Partners have the authority to conduct any business within the scope of the partnerships business At least two owners Liquid capital o Partnership is obligated to pay you off and you can demand to have your money go with you You are guaranteed your interest in the partnership Interest sold without registration o No regulatory hurdles o Cannot defraud someone or mislead them but you dont have any formal process of having to reveal all the material facts Any of the partners can appear in court for the partnership a lawyer is not needed Default form for businesses that are owned by more than one person Two or more persons create a co-owned business without any explicit agreement about their relationship Open to personal liability of all partners for all partnership debts Unlimited personal liability If business cannot pay its debts of any sort, creditors can take from you anything you have that is not judgment proof Most people do not want this risk Partner may dissolve it at any time simply by a statement of his or her express will UPA 31 Settling accounts after If there is nothing left to pay to a creditor and youre sharing losses equally, then you have to kick in the money to share the losses equally o Each party will have the same loss so then non-capital contributing partner will have to pay the capital contributing partner half of his loss Definition UPA 6 Two or more people Co-owners Business for profit Prima Facie Case of Partnership UPA 7 Receipt by a person of a share of profits is prima facie evidence he is a partner in business o Do not have to have received profits yet just have to have agreement to receive profits Can rebut prima facie case if received in payment of: Debt Wages Rent Annuity Interest on a loan For the sale of property Difference between partnership and joint venturers Very similar to a general partnership Uses the same general partnership rules 4

Not partners in a general sense because havent put their businesses together Very limited scope partnership theyre just going to do this one thing together For the purpose of that one thing well treat their relationship as a general partnership Written Partnership Agreements May avoid future disagreements over what arrangement actually was In the absence of a written agreement, the relationship between the partners will be governed by the provisions of the applicable state partnership statute Can clearly identify which property is contributed and which is loaned critical to protect the partners interest in the loaned property Sharing of Profits and Losses Default all partners have equal rights UPA 18 o Share equally in the profits o Share in the losses as you share in the profits Profits of a business may be divided by agreement o Flat percentage basis Percentages for each partner may be specified in the partnership agreement May also issue partnership units to determine the ratio by dividing the number of units owned by that partner by the total number of units outstanding (kind of like shares) If new partners are added, dilution of existing interests occurs automatically without a need to amend the partnership agreement Also gives the ability o Fixed weekly or monthly salary May be treated as a cost and subtracted before the profit is computed Also may be considered an advance to be credited against amount partner is otherwise entitled to after division of profit o Percentage basis Percentages recomputed each year on the basis of average amount invested in business Percentages recomputed each year on the basis of total income, sales or billings by each partner, time devoted to the business, or some other factor o For large partnerships Fixed percentage applied against perhaps 80% of the income Committee of senior partners will allocate remaining 20% among the junior partners for incentive compensation on the basis of productivity, billing or some other factor o Intentionally silent on division of profits Each year partners can work out division of profits by agreement on a mutual acceptable basis Two views o Where one party contributes money and the other contributes services, then in the event of a loss each would lose his own capital one his money and the other his labor o In such a situation, parties have, by their agreement to share equally in profits, agreed that the value of their contributions the money on one hand and the labor on the other were likewise equal Upon loss of both money and labor, the parties have shared equally in the losses Management default rules Nabisco v. Stroud Partnership is the principal in an agency relationship o Partners are agents of the partnership 5

o Agents must carry out the business of principal In deciding what the partnership will do, the majority rules or you need consent of all partners UPA 9 o Majority for an ordinary business decision o Need consent of all partners for extraordinary decisions UPA 18 Addition of a partner is extraordinary The act of a partner binds a partnership unless partner has no authority and the person hes dealing with knows he has no authority UPA 9 o Authority taken away by majority decision All partners start with actual authority to conduct the ordinary business of the partnership UPA 9 o If there is a tie in a vote between the partners on whether to take away the authority of a partner, the tie goes to whoever is attempting to exercise ordinary business authority o When a partnership is not in the business of real estate, selling a major asset is an extraordinary situation and is not ordinary business of the partnership o Many partnership agreement contain a recitation of the business to be carried on o When in the ordinary course of business a partner commits a wrong it is chargeable to the partnership UPA 13 o Where one partner, by fraudulent promises made in a transaction within the scope of the partnership business, obtains money from a third person and misappropriates it, the other partners are liable Rouse v. Pollard UPA 14 o If the transaction is outside the partnership business, the other partners are not liable and they are not bound by a statement of the partner who conducts such transaction that he is acting on behalf of the firm If a third party reasonably believes that the service he seeks is within the ordinary course of business of the profession even if it is unusual for that partnership, the partnership should be held liable Actual authority v. apparent authority Smith v. Dixon o Actual authority Any partner who wants to engage in ordinary business starts with actual authority In an extraordinary business, the partner does not start with any actual authority at all History of situation may give him actual authority o If in the history of the partnership, one partner always did extraordinary business then he may have actual authority An agent must gain actual authority directly from the principal Principal tells the agent you have the authority to do X (express authority) Principal tells the agent you have the authority to do X and that implies there is also authority to do Y and Z (other things that are incident to do X) (express authority and implied authority0 Principal has allowed the agent to do this sort of thing time and again (implied authority) o Apparent authority Look at it from the view of the third party Does it reasonably appear to the third party that the partner had the authority to participate in this transaction? Look at the manifestations (behavior) of the principal 6

Third parties do not have to investigate in order to determine authority but they probably should investigate to determine if the person has authority There may still be apparent authority if a third party is not aware the actual authority of a partner has been removed To determine apparent authority, recourse may frequently be had to past transactions indicating a custom or course of dealing peculiar to the firm in question Giving a title to someone in the company suggests a scope of authority Having a car with the business name on it Having samples of the product to give out No apparent authority just because agent tells third party that he has the authority to do the business must come from the principal An agreement about profit share does not affect the authority a partner has in management o For this to happen there would have to be an agreement that one of the partners would have a separate voice Duties of Partners to Each Other Meinhard v. Salmon UPA 21 IF YOU HEAR THE WORD PUNCTILIA o Defendant always loses o Defendant accused of breach of duty did not meet the very high bar set for him Partners owe each other a fiduciary duty loyalty, faithfulness o Sensitive to morals and feelings of other party o Look out for the other person first and yourself second, if at all o Old standard - Analogy between a partner in business and a partner in marriage Partners ought to take care of one another like spouses should o Now much more like a business relationship Not allowed to: o Appropriate an opportunity for oneself in secrecy and silence When an opportunity comes in the door and is placed in the agents la, that was valuable information (property), for a business can make a difference between profit and loss Cannot be kept by one partner and not share with another To determine if it is a partnership opportunity look at whether it came in the door b/c of the business or whether it was the partner alone who brought it in? Geographic factor o The farther away you get from the business then the easier it will be to say it is the partner who brought it in Broad fiduciary duties under the statute may be modified by an express agreement among the partners Partnership Property If you use partnership money to buy something, it is partnership property What if a partner takes partnership money without consent and gambles with it UPA 21, 22 o If the partner wins he has to share it with the partnership o If the partner loses the partnership does not take on the debt A partner only has a personal property interest in the partnership Partners are co-owners in partnership property UPA 25 o Rights as a co-owner cannot be assigned to someone else 7

As profits accrue, partner has a right to be paid his proportion, on the winding up of business, after the obligations due third persons have been met, he has a right to be paid in cash his share of what remains of the partnership property o This interest can be assigned in whole or fractional part Charging order o Relationship between creditor, debtor and partnership as follows: Charging order may enjoin the members of the partnership from making further disbursements of any kind to the debtor partner Charging order may formally require the members of the partnership to pay to the creditor any amounts which it would otherwise pay to the debtor partner Appointment of a receiver is not indispensable to the collection of the claim out of the debtor partners share Appointed only where he has some useful function to serve, such as maintenance of lawsuit, conduct or sale or representation of computing creditors of debtor partner Debtors interest should be sold if, and only if, the court is convinced that the creditors claim will not be satisfied with reasonable expedition by the less drastic process of diverting the debtors income from the partnership to the payment of the debt Where the partnership or a partner is insolvent o Jingle rule for priority of payment Individual creditors have priority with regard to individual property Partnership creditors have priority with regard to partnership property Execution of a lease o Name of the partnership as the lessee and have all partners signing for the partnership Partnership Accounting Net worth o Value of all the stuff you own minus the value of all the stuff you owe Assets what you own Liabilities what you owe o A-L=NW o A=L+NW Accounting (balance) sheet o Left side assets Cash should be able to accurately figure this out Marketable securities (cash equivalent) Accounts receivable money that is owed to you b/c of the sales of your business Inventory shelf price how much you paid for what youre selling Fixtures anything that will transfer with the real estate b/c its fixed to the land, includes improvements to the property that you have made How much you paid to put fixtures in This is a really soft number b/c the real value of this is not going to be what you paid for it pulling out the stuff and selling it youd probably take a big hit Vehicles bought by the company Tax-related depreciation allowed by tax laws to depreciate some of your assets 8

This is not necessarily the true value could sell it for much more than you put on the books This is not necessarily what you would get if you had to sell everything today artificial o Right side liabilities Accounts payable what you owe other people b/c of the business Note to A (one of the partners) possible for partner to lend money to the partnership Partner could contribute it to capital (could lose the money, just an investor) Partner could lend it and become a creditor of the business Net worth subtract the liability from the assets this gives the partnership equity o Left side has to equal the right side o If the net worth increases over time, they have made a net profit and kept it in the business They could have taken it out of the business and split it but then they probably wouldnt have as many assets Income statement o Sales how much they made in sales o Cost of sales what they had to pay to buy the product they were selling o Gross profit margin difference between sales and cost of sales o Other expenses Advertising Rentals Depreciation any property they have that tax law would allow them to depreciate Salaries Miscellaneous (ordinary business expenses) o Net profit subtract other expenses from gross profit How balance sheets and income statements influence each other o Double entry bookkeeping Any time you make one entry on an accounting statement youre going to have to change another number If affected the income statement then it must have affected the balance sheet If they paid cash for the extra, then they might have less in cash Will have to decrease the net worth also What if they did not pay cash? Would come out of the accounts payable Still would have to adjust the net worth What if partners disagree on whether they want the profits now or want to put them back in the business? o Partners are taxed based on their share of the profits every year even if they dont get the money back out of the business Many partnerships will allow you to take out the money that you will be taxed o Business is not separate entity from partners in terms of taxes they must be taxed based on their share of the profits 9

If there is a loss investor can use the business as a tax shelter, takes away from their total income Capital account o Partners accounting of what theyve put in and got out of this business o Partner with a negative capital account must pay the partnership that amount Restrictive Covenant Limit the professionals right to practice anywhere and access clients anywhere You cannot practice _______ within 50 miles of the partnership You bring someone into the partnership, train them and give them access to all of your tools o Dont want them to go out and open another practice in the area and take away some of your business To determine if valid o What is necessary to protect this business? o What kind of information or training is this person taking with them? o Must be reasonable in distance, in time and in scope Scope the kind of business Distance will vary depending on where you are (city, rural area, etc.) City distance is much smaller than for a rural area Time periods up to 5 years have been regularly upheld For lawyers void as a matter of public policy o Lawyers made this rule as part of professional ethics upheld by Supreme Court For doctors allowed with limitations Addition of a new partner Does not dissolve the partnership and create a new one Does not create the rights of dissolution o Ex: cannot request the winding up of the partnership Inadvertent Partnerships Might enter into a business relationship that looks very close to a partnership, they will disclaim being in a partnership o Statements that no partnership is intended are not conclusive Martin v. Peyton o Intentions are drawn from your actions If you represent someone as a partner, putting his name on the door, etc. UPA 16 o Makes you a partner for all the bad reasons but none of the good reasons Unless you can prove that you are not a partner but this would be difficult o Holding out must be relied upon by the third party This only applies if a third party is attempting to prove there is a partnership partnership by estoppel Smith v. Kelley o Liable to any third party who has relied on the representation This is not the case when a person is trying to prove they are a partner o Must prove an agreement to be partners in that case Partnership by estoppel o Even a reference by one business associate to another as a partner can cause a partnership by estoppel if a third party relies on that representation All partners are jointly and severally personally liable for all debts RUPA 306 If joint liability you have to join all parties in the action Several liability then you can sue some of them, just one of them or all of them This means you can choose several since you could do either Partner that signs a lease with the partnership and then leaves the partnership is still liable for performance under that lease 8182 Maryland Ass v. Sheehan 10

Partners that did not sign the lease and left the partnership before the failure to perform under the lease occurred are not liable o Only the partners that signed the lease or are partners at the time of the failure to perform are liable UPA 17 Let your partners know you are leaving the partnership before you start giving nonconfidential information to another partnership that youre going to Gibs v. Breed, Abbott and Morgan o Anything you know about yourself and your own circumstance is ok to reveal o Already public information is ok to reveal o Client list, if not publicly known, is dangerous territory o Talking to associates and staff Asking them to come to the new partnership with you Depends on notice should tell the rest of the partnership Shouldnt have an unfair advantage in negotiation over those peoples futures Dissolution UPA 31 In partnership, unless something in the writing makes it a more formal process, all it takes is for one of the partners to say I quit Death of a partner is another way to dissolve a partnership UPA 31(d)(4) o If you insure both partners lives, when they die, insurance proceeds go towards their part of the business o If representative of estate of deceased partner so chooses, she can call for the liquidation of the partnership assets, debts paid and share of each partner paid to him in cash Departing partner can ask for their share of the value of the business o Have to liquidate the business or sell it UPA 37 Alternatively, departing partner could instead choose to allow the business to continue and claim as a creditor the value of his interest upon dissolution of the partnership (subordinate to other creditors) UPA 42 o Means he is unaffected by later changes in the value of the partnership determine what is owed to departing partner now and pay it later o Until he is fully paid, the departing partner is entitled to profits since his assets are being used in the conduct of business Unless specified in the partnership agreement, it is the departing partners choice whether to wind up the business or elect to share in profits Options for provisions in the partnership agreement to prevent this o Buy out Partnership agreement could state that anytime a partner departs they have the right to be bought out (paid off) Would not let them force the liquidation of the business If the business is so valuable that you dont have the money to buy out the partner youre back into having to wind up the business o Orderly buy out We will pay you over time We will pay X% first year, X% second year, etc. In the meantime, you will allow us to continue business until you are paid off Partnership is what owes this no excuse for the partners to say we dont owe anything If the partnership cannot pay that money they will be forced into liquidation 11

o Orderly buy out over time using best efforts to pay on time Often times kills the business Period of winding up leads to termination of the partnership o This can be changed by the partnership agreement o Often will say that partnership is not to be wound up and terminated but is to be continued by the remaining partners with the interest of the withdrawing partner being paid off in cash on some basis When there is a partnership for a term certain, the court will not allow a partner to break such contract and dissolve the partnership unless the other partner is not performing on his obligation Collins v. Lewis o Wrongful dissolution UPA 32 A partner who engaged in wrongful conduct may be removed by judicial order and whose financial rights thereafter are reduced by the loss suffered by his wrongful conduct o An expulsion must be bona fide If something affects the trust and personal confidence among partners, the partner who is no longer trusted may be expelled Bohatch v. Butler and Binion Under Sarbanes-Oxley Act this does not include a partner not trusted for whistleblowing When determining the value of the partnership, the market value should be used, not the book value Cauble v. Handler Settling accounts UPA 40 o Order of payment Third party creditors (non-partner creditors) Banks, govt, etc. Subject to agreement with the third party creditor Partners other than for capital and profits (partner creditors) Partners that lend money to their partnership Other than those is debt Partners in respect to capital Investment Partners in respect to profits In order to create any limited liability organization youre going to have to file something General partnership in all respects except that by statute the partners have no personal liability for obligations that exceed the assets of the general partnership Assets they have invested in partnership remain at risk East to convert a GP to an LLP Must complete a few steps, like filing with Sec of State and file an annual report RUPA 1001, 1003 Exception Partners in an LLP have full personal liability for claims arising from their own misconduct o Ex: misrepresentation, fraud, fraudulent transfers, and breach of a duty of full and fair disclosure When signing a contract for the partnership, if partners are going to sign the document the contract must be specific in respect to the liability of the partners Any ambiguity would probably go against the partners Setting accounts after dissolution 12

o Limited Liability Partnership

Partners do not have to personally pay losses, the creditors can only get the money from the business o Unless the creditor has made you sign a personal guarantee Signed away limited liability for that debt o Anytime a partner uses his name for the business he should put LLP afterwards to put the creditors on notice that you are not a general partnership Same with LLC and corporations o Partners do not owe anyone for the losses of the business, even another partner unless you contract differently

o Limited Partnership
Uniform Limited Partnership Act Applicable Provisions o 303 o 304 o 602 o 603 Kind of like a GP Two kinds of partners At least one general partner o General control to run the business o Open to liability o Can incorporate this position At least one limited partner o Limited investment, limited control, limited liability Have to file a certificate of limited partnership to form Capital contributing limited partner could lose his shield of limited liability and inadvertently become a general partner with full liability for all partnership obligations If limited partner participates in the control of the business, he is liable only to persons who transact business with the limited partnership reasonably believing based upon the limited partners conduct that the limited partner is a general partner A limited partner who knowingly permits his name to be used in the name of the limited partnership is liable to creditors who extend credit to the limited partnership without actual knowledge that the limited partner is not a general partner Use of a corporation as the sole general partner of a limited partnership Permitted individuals and entities to be a limited partner even though they are also officers, directors, or shareholders of the corporate general partner If the corporate general partner has nominal assets, the limited partnership becomes a de facto limited liability entity that is similar to a corporation Can be publicly or closely held A limited liability company can be substituted for the corporation If board of directors of corporate general partner have a duty to manage the property of the limited partnership then the other parties who have a claim to that property should have the ability to go after them o Board of directors owe the limited partners a fiduciary duty

o Limited Liability Limited Partnership


LP that gives the general partner limited liability Limited partnership with both general and limited partners but the general partners have the protection of the LLP election Used narrowly Taxation 13

Tax Table Each tax table says if you have taxable income of up to a certain amount you must pay this much money of tax Corporate o Ex: $80,000 tax equals $13,750 + 34% of $5,000 What is the marginal tax rate of this corporation34% $13,750 comes from 15% of $50,000 and 25% of the next $25,000 Individual No partnership tax table partners taxed for partnership business as individuals Marginal Tax Rate Percentage taxed over the low end of the tax bracket youre in When it goes up the more money youre making its progressive When the marginal tax rate goes down the more money youre making its regressive Average Tax Rate Taxes paid divided by taxable income Averages the 15% of $50,000 and 25% of the next $25,000 and 34% of the next $5,000 Basis Amount you paid for property that you bought for investment purposes Stepped up Basis Property acquired by inheritance, generally the fair market value of the assets on the death of the decedent Adjusted Basis Basis of the property plus capital improvements made by the seller, commissions originally paid by the seller, legal costs for defending or perfecting title minus returns of capital, particularly depreciation claimed as tax deduction, depletion, deducted casualty losses, insurance reimbursements and the like Amount Realized Amount you sold it for minus selling expenses Conduit or pass-through taxation Must put their income from the partnership on their own returns Flows through to individual partners Only one tax level entity is not taxed Even if you do not receive any of the profits from the partnership individually you are taxed as if you did o Will work out a deal where partnership pays out taxable income attributable to the partners to cover the tax bill Attempt to minimize taxable income (year end planning) spend the money so that you arent taxed on it If you are going to be making money you might spend some money on the business before the end of the tax year rather than have some big business tax bill Will reduce the ultimate tax bill Check the Box An entity will be classified as a corporation for tax purposes if created under a statute that describes or refers to the entity as incorporated as a corporation, body corporate or body politic or a s joint-stock company or joint stock association o Taxed as a C Corporation or an S Corporation Not classified as a corporation and has at least two members can elect to be classified for tax purposes either as corporation or as a partnership by making election at the time it files its first tax return 14

If entity does not formally elect to be taxed as a corporation it will be taxed as a partnership Entity that has only one member may elect to be taxed as a corporation or it will be taxed as a nothing If entity elects to change classification, may not change its classification back within 5 years without permission of Commissioner

Limited Liability Company


o o Uniform Limited Liability Company Act Adopts more partnership concepts than most existing state LLC statutes Applicable Provisions 101 103 105 106 112 202 203 301 303 404 405 Overview Business organization that takes the best from both corporation and partnership Choice between centralized or decentralized management ULLCA 404 Member managed LLC o Managed more like a general partnership o Members are very much like partners Manager managed LLC o Managed more like a corporation or limited partnership o Managers have principal fiduciary responsibilities Just have to put it in the operating agreement and articles of organization No requirements for internal procedures Forced check-the-box taxation Choose between pass-through taxation or entity taxation Certain entities not eligible for this taxation and must be taxed as corporation o Entities organized under a federal or state statute that refers to the entity as a corporation or as incorporated o Certain foreign entities that are specifically listed in the regulations as per se corporations o Business entities that are taxable as corporations under other provisions of the Internal Revenue Code, such as publicly-traded firms, real estate mortgage investments conduits and regulated investment corporations Removed any incentive to organize a Subchapter S corporation rather than as an LLC Liability ULLCA 303 Limited personal liability Can contract not to have it Life Perpetual life Can contract not to have it Agency 15

Broad agent authority Can contract not to have it Ownership As few as one owners Capital withdrawal You can have it either way Generally it will be an illiquid situation Interest Securities o Not perfectly clear on this question Filing Required Annual registration and fees Must be represented by a lawyer in court Poore v. Fox Hollow Enterprises Dissolution Partnership principles Fictional Entity Cannot suffer mental anguish and distress o Members can suffer mental anguish Can sue for reputational damage Formation Filing Articles of Organization in a govtal office Name of the LLC must contain words or abbreviations indicating that entity is an LLC ULLCA 105 Must be distinguishable on the records of the Sec of state from the name of other registered business names, name reserved or a fictitious name for a foreign company authorized to transact business in the state You can reserve your name in advance ULLCA 106 o Will check it with other names that have been reserved and registered with other business names o You have 120 days before you have to file anything to create the LLC Register you have filed your articles of organization with the Sec of State Principal reason to register is so that foreign organizations (from other states) can come into the state and register their name even if theyre not actually doing business here but they might want to do so later Must maintain a registered office and registered agent for service of process Foreign LLC must obtain a certificate of authority to transact business in another state Can be formed for any lawful purpose ULLCA 112 Articles of Organization ULLCA 203 Affirmative election to be manager-managed must be made here In state statutes usually affirmative election to be member-managed must be made Operating Agreement ULLCA 103 May be oral Basically just a contract between all the members of the LLC agreeing on how the LLC will work Governs the internal relationships of the LLC Operating agreement controls over the articles of organization in case of conflict May not eliminate the implied contractual covenant of good faith and fair dealing When a seller intentionally misrepresents a fact embodied in a contract, that is, when a seller lies public policy will not permit a contractual provision to limit the remedy of the buyer to a capped damage claim - Abry Partners V LP v. F & W Acquisition LLC Membership 16

o o o o o o o

A member must be a person Can be a natural person but can also be any number of business associations or other legal entities o Could be corporations, general partnerships, limited partnerships, regular individuals Other individuals and organizations are participating in those organizations Right to information Emails can be company documents Marie L Kasten v. Doral Dental USA Only emails that are of a business nature and pertinent to the business activity (reasonable standard) Limited liability for all participants whether or not they are active in the management of the business and permits total flexibility in internal management ULLCA 303 Benefits of incorporation without the limitations and rules applied to corporations Formed by filing a document with a state official Primarily replacing general and limited partnerships Closely held partnerships to a much lesser extent Used to hold non-business assets in a spend thrift trust type relationship Owners called members Same roles as a partnership management Fiduciary Duties To LLC and business owners Duty of care Obligation to be well-informed, take time necessary to make decisions and make wellfounded decisions Not a duty to be right, you can make mistakes, you can even be negligent and not be in violation of duty of care Cannot be grossly negligent, criminal, fraudulent, etc. Business judgment rule o If a manager of a business entity applies business judgment in making a decision the court will not second guess that o Were they grossly negligent? Duty of loyalty Self-dealing prohibited Also prohibits any actions that place the fiduciary in a role that competes with the LLC Requires that transactions that pose conflicts of interest be structured and consummated so as to mitigate such conflicts and to protect the party to whom fiduciary duties are owed These duties override any other duties, such as Duty to act in good faith and the duty of complete candor and disclosure o Require fiduciaries to disclose all information that the minority investor would find relevant in making decisions about its investment and all information necessary to make any other information disclosed not misleading When it is decided to sell a company in a change of control transaction, responsibility is to endeavor to secure the highest value reasonably attainable for owners (undertaken with due diligence and good faith) Blackmore Partners LP v. Link Energy LLC For LLC to sign agreement someone would have to sign for it after being authorized to sign for the LLC Elf Atochem North America Inc v. Jaffari and Malek LLC If members sign a contract, the LLC is bound It would be preferable for the LLC to sign but not necessary Treats the LLC more like a partnership in this case In deciding whether to pierce the shield of limited liability and hold the partners responsible, courts have looked to corporate law governing piercing the veil 17

Fiduciary duties on controlling shareholders who act in a way that the court perceives as unfair to minority interests General incorporation statutes anyone who wants to create a corporation can pay a fee, file a document and report back every year Model Business Corporation Act Applicable Provisions 2.01 2.02 2.03 2.04 2.05 4.01 6.01 6.03 6.20 6.21 6.22 6.25 6.26 6.30 6.31 6.40 7.01 7.02 7.04 7.20 shareholder list 7.22 proxies 7.25 quorum 7.28 cumulative/straight voting 7.30 voting trust 7.31 voting agreement 7.32 7.40 7.41 7.42 7.43 7.44 7.45 7.46 8.01 8.04 classification of shareholder 8.05 election of directors 8.06 staggered board 8.20 8.21 8.24 8.25 8.30 8.31 8.33 18

Corporation
o

8.40 8.41 8.60 8.61 8.62 8.63 14.07 Other Claims Against Dissolved Corporations (Successor Liability) 14.30 14.34 15.05 Internal Affairs Doctrine Overview Centralized management Notion that there is some sort of pyramid structure with a management group that runs the show Entity taxation Limited personal liability Perpetual life Could last forever Corporation has own identity and own life separate from the shareholders Discrete agent authority Corporations have titles and positions for people and they spell out to some degree the responsibilities of all the participants One or more owner Illiquid capital As a shareholder in a closely held corporation, unless you have some epical agreement, when you decide to leave and you want your money youre going to have to sell your shares to someone o You might have a problem finding a buyer o In a closely held corporation it is often impossible sell your shares Liquid capital In a publicly held corporation, you have the ability to sell your stock on the market and that makes it even easier than a partnership Interest = securities Reveals all material facts pro and con about the offer and exchange of interest Whenever you offer a single share of stock for sale to another person, no matter who you are or they are, you must register that offering with state and federal govt or find an exemption for that offering o Registration statement o Stock is the classic form of security Filing required Must file the articles of the corporation with the secretary of the state Partnership can be inadvertent, nothing formal must be done Annual registration and fees Must be represented by a lawyer in court Whether a public market exists for ownership interests in the business determines whether the corporation is closely held or publicly held Closely held One or few owners No publicly traded shares Easier for a director, officer or shareholder to incur personal liability Publicly held 19

Hundreds or thousands of owners Mostly incorporated in Delaware The law of the place where the corporation was incorporated applies Corporation has shares that are traded on public securities markets subject to federal regulation Disadvantages in two areas Federal income taxation Mandatory procedural requirements that may increase the cost of operation Corporate existence begins upon the filing of articles of incorporation Results in the creation of a new legal entity, a fictitious person with sole responsibility for its own obligations Provides limited liability for all investors and participants whether active or passive MBCA 6.22 (for shareholder liability) Where to incorporate Business confined to one state incorporate in that state Business across many states Delaware Internal Affairs Doctrine MBCA 15.05 All questions having to do with corporation law are going to be answered by the law of the state in which you are incorporated Delaware is most attractive for publicly traded corporations while closely held corporations tend to incorporate locally Delaware Race of the lax (race to the bottom) o Law is so liberal that it allows corporations and managers to do whatever they want Judiciary has most expertise when it comes to deciding questions of corporation law o Corporations do not like legal risk Two tests on whether Delaware law is good or bad o Tobins Q Compare the book value of the company with its market value How is it using the assets and the debt that it has If the book value is higher than the market value then its not doing so well Delaware companies do better managers are using the assets of the company and the debts of the company wisely o Comparative share prices Delaware companies tend to be more valuable Just because you are a Delaware company you will be making more money from using your shareholders investments than if you were in another state Because under the law of Delaware managers have more freedom to take the risks they think are necessary without being challenged by the shareholders o Battle is often between the management and the shareholders Which group should have more say in the fundamental questions of running the company All the states say that the corporation shall be managed by the board of directors If management gets certain things wrong (breaches duties, wastes assets) then the shareholders can step in and vote them out, sue them, etc. Delaware is more skewed toward management control and limited shareholder voice 20

Managers have more freedom of movement and less threat from shareholders o Shareholders are like mushrooms kept in the dark and fed a lot of manure o What if there is bad management? Sell your stock there will be a market if its publicly traded If more shareholders sell then the stock price will drop lower and lower If the company is successful then more people will buy stock and the price will go up Takeover I could buy the company, kick bad managers out, put in new managers and make money Managers know this they cannot run the company into the ground, must keep value of company up or else they will be replaced There is a market out there that keeps the managers considering best interest of company If you want to buy a company you buy the stock for the company o All you have to do is buy a majority of the stock to take over the company How to change your state of incorporation Dole Food Company Inc, Proxy Statement You incorporate a subsidiary in the state you want to change to (just send in the filing papers and money) o Subsidiary basically has nothing o Vote to merge the original company into the subsidiary o Must get shareholder approval Proxy process MBCA 7.22 Must send out to all of its shareholders to vote for or against the proposal and the proxy statement describes what is being voted on and all of the material information the shareholder would want to have o Shareholders are like sheep tend to do what management suggests they should do Allows you to change the law your corporation is bound by Federal Regulation Securities law Stocks and bonds are the classic securities Regulates companies who buy and sell stock (pretty much everyone) In a publicly held situation it has even more control Requires meaningful and effective ways to vote, anti-fraud provisions to protect the shareholders, takeovers are federally regulated Securities Act of 1933 Truth in Securities Act o Designed to attempt to insure that there is honesty in the process of buying and selling securities Dont want lies and omissions Securities Exchange Act of 1934 o Covered a lot of ground o Created the oversight of the national exchanges o Created the FTC which became SEC o Covered securities fraud of all sorts o Proxy regulation 21

All new publicly traded companies must be registered with the SEC Must file a quarterly report with the SEC all actions and other important things sums everything up Documents available to anyone in the world CEOs, directors, accountants, etc. all personally liable for the inaccuracy of the reports o Initial public offering (IPO) First offering of securities to the public o All sale of stock must be registered Subject to exemptions Private placement o Selling a lot of stock but only to a few people Local offering o Selling a lot of stock but only selling it in one state o Will still have to register with the state Regulation deed Executive Personnel Exemption SEC v. Ralston Purina Co. o Do they have access to the kind and extent of information would exist in a registration statement? If no, then does not apply o Employees are not sophisticated, dont know what questions to ask and not in the position to ask them o Do the prospective purchasers need the kind of protection a registration statement provides? o Applies to all securities Big one that gets lost is an investment contract Investment of money in a common enterprise with the expectation of profit derived solely from the efforts of other people Howie o Common enterprise Smith v. Gross Vertical commonality More normal common enterprise is a corporation Anytime a bunch of people invest in one thing and pool their resources its a horizontal enterprise If you dont realize something is an investment contract then bad things happen o Buyer gets money back no questions asked if you dont register the investment contract If you get it wrong and someone alleges it was intentional then prison could happen o Any time it is a security and violate registration when there is no exemption the investor will get their money back Two theories o Sunshine Theory You can prevent a lot of crime and misbehavior if you force people to act in the open, when youre out in the light (sunshine) Reveal all of your secrets if youre going to sell securities All information available to public Sale and offering has to be completely transparent Will make it easier to hold people liable and well hold you liable if you dont do it out in the open Disclosure-based disclose all facts 22

o o

Efficient Market Theory States securities commissioner will look at offering prospectus and decide for state if this is a good or bad investment if a bad investment the offering would not be allowed in that state Merit review or merit offering Proposed at federal level and rejected Assume that markets are efficient If you pump all the information about a security into the market place then the market will set a reasonable and fair price If there is bad information then people wont want to buy it and the price will be depressed If its good then people will want to buy it and price will go up o Disclosure under sunshine theory will help to create an efficient market by informing the public Sarbanes-Oxley Act Responsibility is to go up the ladder when you discover a company you are representing is doing something illegal Report the matter to the appropriate person in the company and you observe whether anything is done about it o If you dont get a satisfactory response then you go another step up the ladder o If no one listens then you go to the board b/c the president and general counsel have refused to correct the behavior o If the board doesnt listen then you withdraw from representation Any time stock is offered there must be either a registration of that offer with the SEC or an exemption of registration Even if there is exemption sellers must make full disclosures about the stock or else it is securities fraud Three core documents Articles of Incorporation Bylaws Shareholder Agreement Pre-incorporation Transactions Are the owners likely to start acting as a corporation before they create the corporation? If yes, there is some chance, then you should form the corporation sooner rather than later (do not allow pre-incorporation documents) If there is no chance then you should wait until after you have gone over the documents Shares of the future corporation can be sold pre-incorporation Subscription agreements MBCA 6.20 o I promise to buy the stock when the company is incorporated if it is incorporated in the next 6 months o Standing offer for up to 6 months Can bind people to do other things like serve on the board pre-incorporation as well Promoters People involved in organizing a business o Finding investors Fiduciary duties o To other promoters Like you entered into a partnership with each other and your business is promoting a business o If you are acting for the company you cannot get a secret profit off of something you bought for the company 23

If you bought it for yourself before you became a promoter and you are going to sell it to the company you could sell it for a profit o Cannot buy property at inflated price which is worth less If shareholder is upset about this she will have action against promoter b/c promoter took something from the stock that is more than youre allowed Promoter defense no fraud or deception b/c we knew exactly what we were doing, we were the only owners of the company at the time Pre-incorporation contracts MBCA 2.04 If it is a contract there must be parties on both sides of the contract argument is that the promoter is liable signed the contract as the owner Stanley J. How and Assoc. Inc. v. Boss Novation o New contract in which all the parties agree that they will give up their rights under the first contract in exchange for what they receive under the new contract o New contract, new signatures, new corporation signs Adoption o Corporation adopts the contract, architect agrees, corporation accepts benefits of contract and leads architect to believe corporation is liable for the contract Ratification o Corporation can only ratify events or contracts that occurred while the corporation was in existence Did the other party intend to only look to the corporation for payment or did he also look to the promoter for payment? Only liable if knew there was no incorporation but represented that there was joint and severally liable o Knowing actual or constructive knowledge So easy to find out whether or not you have a corporation in place, if you havent talked to secretary of state and you dont have the paper in your hands then you should assume you dont have a corporation in place If you know you dont have a corporation you cannot have a de facto corporation b/c not acting in good faith (see pg. 26) Corporation by estoppel might exist but there is a misrepresentation and that could cause a problem Should not allow a person to take advantage of corporation by estoppel if they knew there was no corporation Three types of equitable corporations Robertson v. Levy o De jure (legal) corporation A corporation that has filed its articles and the articles have been accepted o De facto corporation Was there an incorporation statute allowing incorporation in this situation? Must be able to show you attempted to create the corporation Took some steps created articles and sent them off If gave the articles to the lawyer and the lawyer did not file them then it is not an attempt to file Have to use the corporate name in the transaction Must all happen in good faith o Corporation by estoppel Cranson v. IBM When the plaintiff made a representation to make a contract with the corporation and that they will only look to the corporation for payment then the plaintiff is estopped to the deny the corporation Test Equitable estoppel situation 24

o o o o o

Representation Need someone to rely reasonably on that representation Reliance to their detriment Person making representation is estopped to deny its truth

Formation Corporation comes into being at the time of filing MBCA 2.03 Incorporator Must be a natural person in some states but MBCA allows business entities as well Person that files the Articles of Incorporation No further function past the filing might hold other roles also Articles of Incorporation MBCA 2.02 Purpose clause o A narrow purpose clause is not required but may be necessary if you want to prevent the business from getting involved in anything else o Ultra vires Beyond the powers Entity that took that action did not have the legal power to do that Only people who can bring the action are the shareholders by saying that my corporation is doing something it is not authorized to do All predicated on a narrow purposes clause Modern articles have no purposes clause at all and if you do not have one you are entitled to enter into any lawful business Almost impossible to do anything that is ultra vires Exceptions to the general rule o Charitable giving There is a limit to charitable giving can go beyond powers if they give away too much money (beyond money to reasonably use it in any business sense like tax deductions) Go to the corporations articles is there a purposes clause? If not then very little that is ultra vires If it has a narrow purposes clause there are a lot more things that are ultra vires Name MBCA 4.01 o Must be distinguishable upon the records of the secretary of state from other corporate names o Requires a determination whether the proposed name constituted unfair competition with existing corporations Confusion in an absolute or linguistic sense o Corporation may generally conduct business under an assumed or fictitious name to the same extent that an individual may o Fictitious name Businesses, corporations and others occasionally use names that are not their formal business names Like the name on the sign might not be the real name of the business If thats the case, youre operating a business under a fictitious business name and youre supposed to file a document with your 25

local county to tell them that your formal name is doing business as your fictitious name and here are the owners Purpose if you have a problem with the business then you can find out from a govt source who owns the business and what the name of the corporation really is Protecting your directors from liability MBCA 2.02(b) o In immunity (raincoat provision) Can put in provision in your articles that eliminates liability of managers to corporation or shareholders Breach of duties in most ways certain fiduciary duties cannot ever be breached o In indemnification Put this provision in your articles Corporation will pay for directors damages in a suit o Insurance Directors and officers insurance Insurance policy on directors so the company does not have to foot the bill if there are suits like this Can limit the duration of the corporation Must be registered place of business and a registered agent authorized to accept service of important documents o Your registered agent will preferably be an attorney who will be able to take these things seriously Must authorize one or more classes of shares that have unlimited voting rights, one or more classes of shares that will receive net assets of corporation upon dissolution MBCA 6.01 o These do not have to be the same set of shares, you can have multiple sets of shares that do all of that o Beyond that youre free to design shares as you please o Amending to authorize more shares Must give the initial shareholders more shares Ex: initially 1000 shares, amend articles to authorize 10 million shares, will initially convert those shares into 1 million shares, conversion ratio will be 1000 to 1 o Must have some outstanding MBCA 6.03 Election of Preemptive Rights MBCA 6.30 o When the corporation offers shares they must offer them to all existing shareholders first and they must offer them proportionately o If there are three existing shareholders that offer 1/3 of the shares and the proposal of the corporation is that they will offer 300 new shares, then they must offer 1/3 of the new shares to the 1/3 shareholders first o Gives them the opportunity to preserve their interest o Opt in provision You have to choose to have this by putting it in the articles o Not likely in a publicly held company because there are so many shareholders Would require communicating with every shareholder with that type of stock Creates an expensive layer of communication and complication o Exceptions MBCA 6.30(b) If share is sold for something other than money If share is sold to repay a past debt 26

If there is a flaw in the process of the sale such as coercion or fraud then the whole offering is flawed

Bylaws Internal set of operating rules for the corporation Number of directors o Required to have at least one o Issues about insurance if lawyer is director Dont be a director in the corporation if you are the lawyer for the corporation o Tie-breaker directors are not a good idea if they are brought in solely to be a tiebreaker They have all the power in the situation o If only two directors instead of bringing in a tie-breaker they should just act as partners o If you have fewer than 50 shareholders you can agree to eliminate the board of directors Issue if only two shareholders and 50/50 split then will just have to agree, if a 51/49 split then the person with the 51% interest makes all the decisions Need to have an annual meeting with the date set MBCA 7.01 Board meeting dates Vacancies o Ex: 5 directors and all are shareholders, one dies now 4 directors and a vacancy o How do you fill the vacancy? Who will make that decision? Shareholder or board? Can choose in the bylaws Shareholder agreement What happens when a shareholder departs the company Getting Started Organizational meeting MBCA 2.05 Determine who the initial directors will be and who they will name as officers Bring in some capital sell the stock o If you have a partnership already in existence before you incorporate, will transfer interest from partnership and will use it as consideration for stock in company that will be valued Put a notice in the local paper of general circulation where your registered office is within a day of filing the corporation o Notice that you are not a limited liability company o Publish for two weeks running o Newspapers like this b/c legal notices are a source of income o If you dont do it theres no penalty There doesnt have to be an organizational meeting lawyer draws up minutes and the people that were supposed to be there will consent to those minutes same effect Business is conducted in the corporations name Corporation itself enters into contracts, borrows money, sued and may be sued in its own name and otherwise conducts business much as though if it were a real person Assets are subject to seizure by corporate creditors and the shares of stock of the corporation are subject to seizure by personal creditors of the partners Signature lines Signature line should have name of corporation underneath it and the state the corporation was incorporated in and then a byline with the name of the signator there and the title of the signator Anything that departs from this is dangerous 27

o o

If you dont put By: Signator and just put signators name the signator will be held liable as well as the corporation If you put By: Signator (personal) Corporation liable, signator is not liable b/c By: holds more weight than (personal) Authorized Signature under Signators name with no By: Signed as an individual could mean authorized signature of himself, corporation and signator both liable

Three tiers Shareholders who are traditionally viewed as the ultimate owners of the enterprise The board of directors, composed of the managers of the corporations affairs Board of directors chooses who to fire and hire in a close corporation The officers who act for the corporation to implement the decisions of the directors A single individual may simultaneously act as an officer, a director and a shareholder Fiduciary duties Shareholders have no fiduciary duties They are meant to be selfish Those who are running the company have fiduciary duties owed to the company and indirectly that means to the shareholders Liability Limited liability for those involved in a corporation Shareholder may be liable if the shareholder agrees through some kind of contractual promise (such as articles of incorporation) to be liable Generally speaking, a corporation would not voluntarily have this provision No personal liability for losses suffered by creditors for ineffective management of the business Personal Liability MBCA 6.22 If you commit a tort could be liable If you sign a contract could be liable Liable for the consideration you were supposed to pay for your shares MBCA 6.22 Corporate Veil Equitable, judge made doctrine Can the owners of those corporations and occasionally those that act for it in some sort of way be liable for its contract and tort debts? Purposes o Prevent fraud o Achieve equality There are reasons to ignore the corporation (disregard it) and pierce the veil List of factors that courts look at when theyre trying to decide whether to pierce the veil o Fraud Misrepresentation (which is an element of fraud) o Undercapitalization Baatz v. Arrow Bar What capital would be reasonably needed to pay the debts the corporation would expect to have Also think about what kind of problems may happen and how much money would need to be dealt with Ex: a company that operates a swimming pool someone could drown a company should buy insurance to prepare for this company that neither has insurance nor enough cash to deal with a suit, this could be a situation of undercapitalization Insurance can be used to suggest that you are not undercapitalized for tort liability but not for contract liability Radaszewski v. Telecom Corp. 28

Should have $1 of equity for $10 of outside debt Should have $1 of equity for every $3 or $4 of inside debt This is still pretty thin o Corporate formalities Board and stockholder meetings as required by statute Proper order of payment of bills Corporate records If dont observe corporate formalities then weighs in favor of piercing the corporate veil o Comingling (of assets) Dewitt Truck Brokers v. Flemming Fruit Co. Relates to formalities Occurs when you treat corporation funds as if theyre your personal funds (or vice versa) o Instrumentality and alter-ego doctrines Agency concepts In a close corporation, the shareholders are entitled to run the show, but it must be done in an appropriate way The shareholders should be running the show for the benefit of the corporation and do whats in the best interest of the company Parent-subsidiary situation o Parent company is the shareholder of the subsidiary Wholly owned subsidiary parent is the only shareholder If not then there is a parent corporation and there are some other shareholders out there that hold stock in the subsidiary company o When there is a wholly owned subsidiary (corporation owns 100% of the stock of the subsidiary) and the parent corporation does not put a lot of money into the subsidiary, to pierce the corporate veil and get any money out of it you would have to pierce the veil of the parent corporation as well as long as the subsidiary is not undercapitalized Bartle v. Home Owners o How do you hold the parent company liable for the subsidiaries? Shareholders will choose the board of directors shareholder is the parent company As the only shareholder, the parent will elect all of the directors If you are a smart parent company you will pick people that you think will best serve the interest of not only the subsidiary but also the parent company Creates a potential conflict of interest If you were one of those directors, you are a significant employee of the parent company and youre elected to be on the subsidiary company board o Youre going to be potentially thinking what is in the best interest of the subsidiary but at the same time you will say youre here b/c the parent company put you here, should think about their best interest as well Situation where subsidiary has contract with the parent o Parent is not performing on the contract o Somebody might say that subsidiary needs to sue the parent, but another person may say that theyre sitting there b/c parent put them there When a wholly owned subsidiary has a board that was completely elected by the parent corporation a plaintiff could argue: 29

Control o Complete domination o Control must have been used to commit the wrong o Proximate cause of injury Alter-ego Instrumentality This parent is operating this subsidiary as an agent for the parent o Just playing out the parents business with no mind of its own o Should not be respected as a separate business Must prove parent and subsidiary are the same economic entity Fletcher v. Atex Inc. o Did the subsidiary board of directors act on its own? Look at board records and minutes Interlocking directors when some of the people on the subsidiary board sit on the parent board as well o Did the subsidiary file its own tax returns? Not a significant factor Subsidiaries and parents will often consolidate their tax returns and file one return for the whole enterprise o Did subsidiary have its own employees and management that were responsible for day-to-day business? o Cash management system When money comes into the subsidiary where does it go? Does it go to the parent company? If it comes from the parent company it looks suspicious but is not a problem There are corporate economies that can be had if they aggregate all of the financial affairs in one spot for the corporate family Parent becomes a bank for all of the subsidiaries Money is specifically accounted for whose is whose and they always know where it is and what they have but the parent is maintaining the account o Not easy must prove that subsidiary doesnt make any decisions without contacting the parent company first o Can translate this to a close corporation situation Company is owned by one shareholder that is not the parent company but instead is a person Corporation created in your name Were you making decisions for yourself or were you making decisions based on business of corporation Comingling is the big problem Did you have meetings? Did you keep records? Whether parent and subsidiary have an unhealthy relationship if we find out the subsidiary never has enough money to operate individually 30

Undercapitalization Parent left the subsidiary unable to conduct its business responsibly and fairness would suggest you can pierce the corporate veil of the parent company o When a company is owned by one shareholder and the corporation is created in the shareholders name Were you making decisions for yourself or were you making decisions based on the business of the corporation? Comingling is the big problem Did you have meetings, did you keep records? Not legitimate unemployment if you have control over your own employment youre self-employed you can call the shots of when youll work or be laid off Enterprise Liability Related companies can you pierce one companys veil to get to anothers or can you throw them all into a group and pierce them all? o Must show undercapitalization in all the subsidiaries McCloskey v. Carlton not successful o Taxi cab case example in class Scott Brothers v. Warren successful o Corporation loaned money to P at time when company was insolvent o Continued to draw salaries after company was insolvent o Ran 3 companies as one unit using same booking and trade name and didnt tell creditors there were three different companies o Sold assets for lump sum figure and didnt allocate proceeds to the three companies aggregated them Reverse Veil Pierce Cargill Inc. v. Hedge Very unusual concept People who could ordinarily say there is a legitimate corporation and Im a shareholder are now saying please disregard that I have a corporation and treat me as an individual Homestead Exemption o Allows occupants of the actual homestead who basically might lose it to exempt the homestead itself from judgment o If its under a corporation they have no standing under the Homestead Exemption its not their house, its the corporations house o Courts will often let disregard the corporation and get the exemption b/c court sympathizes with farm problems o Equity principle Statutory Veil Pierce When corporation commits certain kinds of acts (e.g. polluting) its owners can be liable for cleaning that up o Particularly environmental law (e.g. CERCLA) o Super funds environmental cleanups If you are an owner or an operator of an environmental disaster (big or little) something that requires clean up, youre obligated to clean it up, if you dont clean it up the govt can step in and clean it up for you and send you the bill If you can show a parent company or related enterprise had control or influence on the site you can hold that person liable also no need to pierce the veil 31

Deep Rock Doctrine Pepper v. Litton Type of bankruptcy notion Can take the money that went to the owner and force it back into the company to be paid to third party creditors Not piercing the veil They think there is a principal here where his claim should be subordinate to or at least not higher priority than third party creditors Only have to give back the money that was paid out v. piercing where the owner will have to pay what the third party creditors are owed with no limit Successor Liability MBCA 14.07 Successor when a company takes over the operations of, is running the business of another company that is no longer in business Two ways to create a successor company o Shareholders decide they no longer want to own the company and they sell all their stock to a third party In that case the company is the same, its not really a successor company Same company but has changed ownership Nothing changes with creditors and liabilities same legal entity o Third party approaches company about buying the assets of the company MBCA 14.07 Creates her own company and uses the assets she purchased to put in her new company The other company dissolves Claim can be made against old dissolved company and may have to pay that claim on a prorated basis (depends on how many shares you had in the original company) Would not have to pay any more than you received from the company Reconstitute the company and assets received from it and pay the debt from that Most states hold not liable unless some things are true Nissen Corp. v. Miller o Agreement to assume liabilities express or implied o If successor company just merged the first company into it (first company still in existence in a sense) then everything comes with it o If successor is just a reincarnation of the first company First company seems to disappear and dissolve but the same people are in the second company and attempt to ignore liabilities of the first company Unfair to allow them to create a new corporation to get rid of the liabilities of the first company o Continuity of enterprise Business remains the same but ownership and management involved in it do not, there is no liability What if the new owner is the only change that paid fair price for the company Policy o Different company not responsible for the behavior that came before it o If successor company has to pay the judgment, he is innocent of any problems she didnt create that, she just bought the assets and as long as she paid a fair price for them then should not be liable o People would never buy companies or assets b/c wouldnt know what liabilities they were getting Insurance is the answer! 32

Capitalization In the organizational stage where we have to bring in capital to start running the business Two basic methods Debt o Borrowed so its a liability o Loans o Debt securities Bonds Like stock but not People buy them and the corporation promises them that they will receive a return on the purchase and at some date certain they will get all their money back Bond is secured o Bondholders will get property if not paid Debentures Promissory note promise to pay If you cant do it then no underlying property indentured to it Junk bonds o Promises to pay and very junior not good enough to have a good credit rating (quality of debt is poor) and corporation will have to pay you insane interest rates b/c not likely to get your money back o Where do you get it? Bank Person Equity o Sell stock Common Most people choose this Voting rights Dividends o After the preferred shares o Some expectation of dividends Liquidation rights o Youre the last one in line and if theres anything left over youll be there to take it Payment o Can sell common stock for cash, property or promissory note o Anything that would be considered consideration under contract would be legitimate for purchasing shares MBCA 6.21 If the shares are sold for something other than cash or a cash equivalent quorum approval of the shareholders is required If the voting power of the shares sold would compromise more than 20% of the total voting power then you must get approval by a quorum of the shareholders Preferred No control (non-voting) 33

Dividend preferences o Fixed dividend dollar amount or percentage of the face value of the stock Liquidation preference o First in line among stockholders when the company is liquidated o Would not necessarily be first in line among all people (e.g. creditors) Interested in money and a return on your money and some security in getting it back if things dont go well but not concerned with control over the company Participating o One you are paid your fixed dividend fully, you get to stand in line with the common share people and act like you have common stock o You have a fixed dividend (can only be a certain amount) in your preferred stock so common shareholders might get more dividends if the company is making massive profits o To make it more attractive you get a first bite at the dividend apple (fixed) and youll get a second bite at the dividend apple with the common shareholders Cumulative o If the company could not pay the dividends one year or only paid part of the dividends, will get that amount you werent paid on top of the regular amount next year Must get that before the common shareholders get anything o If non-cumulative tough luck you just dont get that money Redeemable MBCA 6.31 o Redeemable at some value (usually face value) of the stock at the option of the corporation o You might be a content stockholder and then the corporation comes along and says were going to buy your stock back from you and the corporation can force the repurchase of the stock o Corporation will do this if the company is paying out so many dividends that the common shareholders decide they dont want to do that anymore and will buy back the stock Convertible o Convertible to common shares o Can occur in various kinds of securities o Preferred stocks have preference and can convert to something with lower preferences There will be a ration 2:1 or something like that 2 preferred shares to 1 common share o Usually only convertible if not participating o Once youve converted you cant go back 34

Other Can design your own kind of stock as long as you have: o Must be some stock that votes o Must be some stock there upon liquidation to be transferred the residual assets of the company, if any Can put in your articles that dividends will only be paid under certain circumstances Ex: if we make X number of dollars in profits or if we only have X amount of debt o If the articles dont say what kind of shares the assumption will be common If you want to have two classes of shares you have to specify you want two classes (common and preferred) in your articles Cannot only have preferred stock if it is non-voting since some stock must have voting rights Thin Capital and Leverage v. Undercapitalization Leverage o Using other peoples money to make money e.g. borrowing at 3% and investing somewhere you will get a 10% return you keep that 7% o If you use someone elses money you might get a bigger return on the money you put in Not a lot of capital in the business mostly borrowed money a lot of leverage You attempt to get a higher return when you use someone elses money Whatever you dont pay in interest rates the owners keep If you put your own money in it youre not using anyone elses money to your advantage o No leverage break Make sure youre not so thinly capitalized that someone can argue youre undercapitalized Liquidity v. Stability Liquidity o Can you change the interest in your company (stock) into cash easily? o Closely held corporation is not liquid Possible interested buyers are other owners, the company itself, the public might be interested depending on how the company is doing but a lot of people probably wont o Not liquid unless company itself puts in shareholder agreement that if a shareholder wants to cash out then the company will buy back the stock if it can This causes instability in the company Hurts the company to have to pay you off for your stock Creates instability if theres liquidity o This doesnt happen with publicly traded stocks b/c other people are buyer, there is a market for the company stock, and the company isnt cashing you out, other people are taking your stock from you Asset protection v. Outside capital Preferences If you take all the good features for you and then you want to raise money outside youre not going to be able to attract good investors o Outside investors wouldnt go for you being paid off before they are Control v. Outside capital Dont have to issue the number of shares that you authorized (thats just the maximum number) If youre on the board you have a right to all corporate information and you have control over decisions made 35

o If youre not on the board youre only authorized for public information Outside investors have the choice to buy common stock or debt o Common stock Risk involved but you also get the rewards if the company does well As long as youre candid about the company you can sell the stock for whatever the market will bring Tell the truth or dont say anything at all Will shift the control possibly to outside investors o Debt If you dont want as risky an investment then you will lend money to the corporation and become a creditor better chance of getting your money back but less of a reward if the company does well Stock having par value Not that common anymore If you choose to have par value stock youre assigning the stock an arbitrary value o Means nothing o Suggested that it has a psychological effect that people will think its worth more than it might be o You would set the par value in your articles o You may not initially sell it for less than the arbitrary amount you decided on If you do its called watered stock Selling it for below what its really worth Whats the problem with this? The person you sell watered stock to will become responsible to pay the remaining amount Can sell it for more but not less o When it is sold later, par value is not applicable, it is only applicable when the corporation sells the shares When it is in the hands of the investors they can do whatever they want Selling stock Must register as a public offering with the SEC or find an exemption that applies to this offer of stock This must be done every single time the company or a shareholder sells its stock If this isnt done then anyone who bought the stock has the right to rescind the transaction o Become a guarantor of the value of the stock at that price Most of this happens when they didnt realize they had to register, if they knew they were supposed to register then its an issue of securities fraud o You can get your money back and other kinds of remedies as well Subscriptions MBCA 6.20 Subscribes to the amount of the stock youre planning on being issued Irrevocable for six months If corporation is formed then you have to put up whatever you promised Can be sued for violating subscription agreement Dividends Distribution MBCA 1.40(6), MBCA 6.40 Money is going out of the corporation to the shareholders b/c they are shareholders and the corporation is getting nothing of value back Getting the shares back is nothing of value to the corporation itself (redemption) This covers dividends and redemption MBCA 6.31

36

You need a shareholder agreement or provisions in the articles to protect against dividends not being declared unless there are some special facts If an adequate corporate surplus is available for the purpose, directors may not withhold the declaration of dividends in bad faith - Gottfried A corporate surplus that is not used for dividend pay outs is not in bad faith o Could pay down debt, expand the business, hire more people, etc. with that money Evidence of bad faith o Nuisance value Majority thinks that minority shareholders are a pain in the neck will buy the shares back just to get rid of them This would be evidence of bad faith o Exclusion of minority from employment Board makes the decision not to hire them This is not an illegal decision to not hire a shareholder Ordinary decision of the board business decision of the company and that is protected by the business judgment rule o Will not second guess business judgments unless there was some serious fault with the process used to make that judgment i.e. bad faith o Majority paying themselves excessive salaries o Not paying dividends to protect their tax status You need pretty much all the factors to have a snow balls chance in hell to win If you have the wear with all to pay dividends you should distribute them unless you have a good faith reason not to (floor) Dodge v. Ford Motor Co. Not hard to have a good faith reason Can be a legitimate business judgment about the use of the money that you can defend o E.g. reinvestment in the company, other investments, paying down debt, etc. Ceiling - Cannot pay out dividends if: - MBCA 6.40(c) Equity insolvency o You cant pay your debts as they come due Balance sheet insolvency o Would liabilities then exceed assets? Board makes decision on whether to distribute dividends If make an unlawful distribution, board can be held liable MBCA 8.33 Exit Strategy Valuation of Shares (This is also the ceiling amount for dividends) Book value MBCA 6.40 o Had an orderly sale of assets and paid off liabilities and divided up the money to determine how much each share would be worth Common share equity amount plus earned surplus divided by the number of shares Earned surplus is retained earnings o Past profits that weve left in the company by not distributing it Did something with it that is now under the assets o Very low conservative price o Usually sells for higher than book share o Two tests to be sure that value isnt too much Balance sheet insolvency test If you made this payment would you put yourself into balance sheet insolvency? 37

Take money you give to retiring shareholders and respect the preference of the preferred shareholders and then compare assets and liabilities Are assets less than liabilities? Can you pay your debts as they come due after you pay out the money to the board member? Wont have to be able to answer this When buying back minority shareholders shares they are often times discounted compared the majority premium (control premium) o If youre going to get control by buying the shares then youll pay a lot more o If youre not then you will get a lot less Buying back stock solidifies control of directors b/c there are fewer shares and they will have a higher percentage of the company When a minority shareholder is mad about the valuation of his shares compared to the valuation of a majority shareholders shares, the minority shareholder will argue that the corporation should be looked at an incorporated partnership - Donahue o Incorporated partnership look at the corporation as a partnership o If it were a partnership and one partner decided to leave then the partnership would dissolve and he would have the right to take with him the value of his interest This will often give a minority shareholder a better deal than they would under valuation of shares This will also change the fiduciary duties owed to the other owners of the corporation Must determine if everyone in the picture was treated fairly o If create a market for one owners stock then must create the same market for another owners stock Must have a very very good business reason not to The bigger the close corporation gets, the harder it is to argue for an incorporated partnership Illiquidity of a minority shareholders interest o To resolve this issue at the outset Put something in the articles In our company, any shareholder who wants to leave or be cashed out can be cashed out and put a formula Shareholder agreement Repurchase agreement Might have limitations on the liquidity of shares o Time limit o No cash o Liquidity can be death to a company Especially if a majority shareholder wants to cash out If you take your interest of the company with you then the company will often have to liquidate to pay you off o Could create an orderly exit plan so that the departing shareholder cannot take all of the money at one time o Company can buy life insurance on one another so then the insurance proceeds will fund the buy out of a deceased shareholder Dividends v. Salary If company makes profits youd want dividends paid Company would rather pay you salary b/c then there isnt double taxation and it can be deducted as a business expense 38

o Recipient only has to pay tax on salary o Both have to pay taxes on dividends Bonuses o As the approach to year end of tax year, the company will sit down with the accountants and do some year end planning o If were going to make money this year we can use it and put it back in the company or pay it out in bonuses o Extra salaries justified by the fact that you had a successful year Control and Management Board of directors MBCA 8.01 Oversight responsibilities Can still be a passive investor and be on the board Need at least one director if youre going to have a board of directors How do you ensure you will be elected every year o Contract for it o Could be under the shareholders agreement o Could be a stand alone contract called a voting agreement MBCA 7.31 You will use your best efforts to make sure the group of you are elected to the board every year Can contract to give away your voting rights these are property rights What if you die, what would happen to your interest? o Your shares are personal property so they will pass to your estate if you die o They end up in the hands of some relative probably Would become a minority shareholder in the company Becomes easy to freeze someone out at that point b/c the voting agreement wouldnt apply to them unless you specifically contracted for that Directors appoint officers MBCA 8.40 Officers MBCA 8.41 Officers hire other agents President, CEO, etc. Voting agreement MBCA 7.31 Use best efforts to make sure the group of you are elected to the board every year Public policy in these situations McQuade v. Stoneham (overruled by Galler v. Galler) o MBCA 8.01 directors control Directors are to exercise good faith, loyalty, disinterestedness and independence o THE OLD LAW If there is a contract saying you have to think and act in a certain way then they have constrained their independence Ex: if it were in the best interest to fire a director and the board was constrained by a voting agreement o If all directors were part of the agreement then it must likely would be upheld Directors jobs MBCA 8.01(c) o Not an exhaustive list LAW NOW MBCA 8.01(b) subject to an agreement authorized under MBCA 7.32 o MBCA 7.32 shareholder agreement Can eliminate the board of directors in shareholder agreement If you vote as shareholders you will vote according to the shares you own in the company o Different levels of voting power

39

Can keep a board and say that certain decisions will be made by board and certain will be made by shareholders What kinds of distributions and dividends will be made and when Who will be directors or officers for the corporation, how we will choose them, how long they will be in office, etc. Use of weighted voting rights Some shares worth more than others when it comes to voting rights Director proxies Get someone else to vote for you With shareholders this is quite common in publicly held companies If I cant make it to the meeting, I can give my proxy as a director to someone else o This is very strange if we have rules about how directors as supposed to vote Require dissolution of corporation at request of one or more shareholders or upon the occurrence of some event Everyone who is a shareholder in the corporation must sign onto the shareholder agreement You can have individualized separate voting agreements that not all shareholders participate in but its when you invade the boards powers you must have all shareholders on board New Shareholders Agreement must be noticed on the share certificate MBCA 6.25 o Pull any outstanding share agreements and add the notice to them o If no share certificate must be included in the information packet sent out to new shareholders MBCA 6.26 Should alluded to in the articles Last for 10 years unless some other provision o If the company goes public MBCA 7.32 doesnt work If one party decides to breach the agreement then you should have a mechanism in there that could be specially enforced Ringling Bros. o If you dont have an enforcement mechanism look to MBCA 7.31 Voting Trust MBCA 7.30 Test o Was there the transfer of the title of the stock? o Did the transfer also include the right to vote that stock? If you really want to solidify the voting groups power this is a good option Instead of being a contract concept its actually a property concept Transfer your shares to a trust and get the other people youd like involved to transfer their shares to the trust and then the trust (controlled by a trustee votes the shares Identify a trustee that has the power to vote those shares for the remainder of the trust You transfer your shares, not just voting rights and the trust will send you your dividends Trust is now listed as the owner of the shares, copy of trust agreement must be given to corporation Lasts for no more than 10 years Government takeover of the company o Common shareholders will deposit all shares into a voting trust that the govt will be the trustee of 40

Until certain goals are met the govt will say who the directors of the company are When meet those goals the voting trust will dissolve and the corporation will go back to the way it was Reasons to do this o Bankruptcy and reorganization situations Baltimore Transit Company Pre-bankruptcy two kinds of creditors First priority lien creditors (secured creditors) o Owned property Unsecured creditors When go into bankruptcy, secured creditors are going to grab property and sell it to pay off whats owed Property is not worth whats needed to cover the debt Might get paid fully if attempt to keep company going and get it back on its feet and pay back in the usual way Unsecured creditors are screwed b/c no property to sell and probably not enough money in company to get paid Secured creditors have a lot of clout Wont grab the property and sell it if you let us take over the company Will put stock in voting trust and trustees will vote the directors o Trustees are secured creditors o Locked control of their company in the secured creditors Voting trust certificates are like stock can be sold to other people People who get to determine whether to renew the voting trust are the people that own the voting trust certificates o Transfer of control in a family held business You have started your own business (your family), you recognize that at any point you could die, youd like your children to continue the business but they are young and if they get your stock they might have the theoretical ability to control it but you dont want them to, you would create testamentary voting trust in your will transfer your shares to the trust and name a trustee to run the business until your children are old enough to take over Could also be that youre dealing with adult children you dont trust very much Reasons not to do this o Not well regarded in history or law b/c used for detrimental purposes for the company and people involved Doesnt allow flexibility of the company changes Must have an annual meeting for the shareholders MBCA 7.01 Principally to elect the directors Close corporation will keep records of who its shareholders are o Who has the title to the shares When you use an investment firm they keep all the records of your stock and everything but you have all the rights Proxy contest MBCA 7.22 Its a big political fight o You have two slates of prospective directors one put up by the company and one put up by a shareholder saying they dont like the way the corporation is running with enough shares to have some clout here 41

o o

Both sides are soliciting proxies from shareholders to get the other shareholders to vote for their people Proxy form o Corporation must go to record list of shareholders o Record owners really arent the real owners in most cases a lot of times its a brokerage firm that has a client who wanted the stock o Form will be sent to the list of shareholders Who gets to vote? o In a big company the record list will change almost daily The list of the beneficial owners under the record owners will definitely change o Set a record date that will decide which people will get to vote Anyone that comes in later on after that date doesnt get to vote Cant be more than 70 days prior to the meeting If you are not on the record list of the company as of the record date you can still vote if you receive a proxy card from the record owner at the time it was sent out or you have to have an indication of the right to own the stock Whoever is designated to make these decisions at the shareholder meeting has a lot of discretion in what to do with those decisions o Courts arent supposed to interfere in the middle of a meeting, theyre supposed to wait until the end of the meeting to get everything unraveled Requirements for a lawful meeting o Lawfully called meeting? Was notice sent out on time? Did it have the right form? Did it go to the right people? o Quorum? MBCA 7.25 A majority of the shares authorized to vote default rule Could have a super majority quorum rule or a sub majority quorum rule if specified in the articles Count shares and not shareholders Majority of the shares could be 10 shares in the corporation but we need to know how many shares each person has o Lawful approval of issue Needs majority of the shares of the represented quorum If you do not reach this then you cannot make the decision Election of Directors MBCA 8.05 Typically requires a majority but in some circumstances a plurality Two kinds of voting system MBCA 7.28 o Straight voting Stancil v. Bruce Stancil Refrigeration MBCA 7.28 You can vote the number of shares you have for up to the number of director seats that are open Say there are 3 directors spots open you can vote your shares for the first one, your shares for the second one and your shares for the third one If the shares are split 50-50 and each has their slate of directors then it will always end in a tie If one side has more shares than the other, that side will always win You cannot vote for one candidate any more shares than you own o Cumulative voting MBCA 7.28 You need to elect to have cumulative voting 42

You can vote your total number of potential votes (shares x # of directors) in any way you want to Formula S/(D+1) +1 = n S equals total number of shares voting (not just your shares, all of the shares), and D equals number of directors to be elected This equation gives you the total number of votes needed to elect one director Ex: 25000 / (3+1) + 1 = 6251 To determine how many directors you can elect take your shares x # of directors being elected and divide it by the number of votes required to elect one director Gives minority shareholders the chance to gain representation were in straight voting they would not have this opportunity Directors get all information they otherwise wouldnt have allows them the chance to be elected to the board Good way to get the minority shareholders some control Classified (Staggered) board MBCA 8.06 You can divide the board into 2 or 3 classes of directors and every year some of those seats will be up for election When the number of directors decrease and the shareholders stay the same then the minority shareholders will need more votes to elect a shareholder o Classifying (staggering) the board will make it more difficult for any given shareholder or shareholder group to elect a director Reasons to allow classification o Gives more stability to the board, board members dont have to be so nervous about reelection You can make it so that certain classes of shares get to select certain classes of directors MBCA 8.04 Should not put your lawyer on the board Lehrman v. Cohen As soon as lawyer casts first tie breaking vote you have made enemies with half the board and they will try to make sure you dont represent the company anymore Perfectly lawful to do this even if stupid When a director leaves the board it creates a vacancy To fill a vacancy MBCA 8.10 o Shareholders may fill vacancy or o Board of directors may fill vacancy Wrinkles Deadlock MBCA 8.05 o Gearing v. Kelly o Somethings occurred that prevents movement or action There is usually an even split in power in some way o If you cannot elect and qualify new directors then the incumbents stay on the board Lasts until the deadlock is broken o Sue for involuntary judicial dissolution MBCA 14.30 Kills the company Not automatic Court has discretion Courts dont like to dissolve companies

43

Tend not to if the company is healthy and making profits In re Radom & Neidorff Inc. Might order it anyway because a buyout may happen o Once dissolution is order by the court the shareholders can step up and say they dont want dissolution, they want to keep the company going and they will buyout the other shareholders MBCA 14.34 Must be a fair value not necessarily an appraisal value You would negotiate this price Court may step in and assist in that process if cant come to an agreement Court has power to order a buy out without ordering a dissolution first Davis v. Sheerin o Oppression Unsavory behavior to illegality Unfair treatment to something criminal o Illegality o Waste Giving assets or shares of the corporation away o Deadlock o All lead to dissolution or buy out How do you prevent this? Vacancy in one faction only allow that faction to vote for it Make two classes of shares identical except each only gets to elect 2 directors If they want to sell their shares when they sell them to someone else it comes along with this right to elect the directors

Authority Corporation by its very nature leads to a lot of agency issues because the corporation is a fictional person Must act through agents If we identify who the agents of the corporation are and what their scope of authority is we have a better sense of how the corporation should act and what procedural form those actions should be taken under Directors are most likely not agents unless they hold some other position Brains of the outfit Brain cannot carry out any action that is apparent to the outside world o Decide what action will be carried out and by whom o MBCA 8.01 Every other position in the company from president down to the lowest rung on the ladder of the corporation are all potentially agents of the corporation There are also people outside that are agents of a special sort like independent contractors Corporation seeking a loan Some officer in the corporation decides that a loan is needed Decision to go seek the loan would be decided by the board o Since this is an extraordinary decision it has to go through the board and an officer cannot just decide to go get a loan Documents involved in this transaction o Promissory note signed by an authorized agent for the corporation 44

Personal guarantee possibly Authorizing resolution From board of directors saying they authorize the agent to negotiate the loan This document will be signed by another officer with the corporate seal affixed Possible there was a fraud there still Document can be forged If everything looks legit but is not then the corporation still has to pay because of apparent authority o Company was defrauded by its own people Procedural Matters Board meetings MBCA 8.20 o Boards are supposed to manage the company but it doesnt say how often they have to meet to carry out that function In a close corporation you will find sometimes that they never call a formal meeting If you do not hold a meeting, there must be unanimous consent by the directors for the company to take action MBCA 8.21 If youre going to hold a meeting you need a quorum majority of the directors MBCA 8.24 How do you get power as a director? o You meet as a board and the board has power, individual directors dont have power collegiality principal o Telephonic rule as long as all directors can hear and be heard then ok Action by consent MBCA 8.21 o Requires unanimous consent can be modified by rule o If a director wants to take action without a meeting they must send around a consent form and it must be unanimous decision to take action by board Officers President or CEO usually owns a lot of stock o When he takes action which hat is he wearing (stockholder officer and director)? Will dictate his duties and authority o Senior most official in the company Titles are not necessarily the kinds of things that have clear definitions o CEO chief executive officer o CFO chief financial officer o CIO chief information officer Presidential authority o Historically No authority Board had all authority and if board does not delegate authority to president then he has none Problem civilians think when theyre dealing with the president of the company that theyre dealing with the person with all the authority for the company o Modernization of law Lee v. Jenkin Bros. Co. Third parties may reasonably believe that the president can make contracts, hire, fire and engage in business arrangements for the company Presidents now have the implied authority to enter into ordinary business contracts 45

o o

President has the authority to enter into ordinary business contracts without going to the board for specific approval If board is uncomfortable with that law, then they can communicate with the president in some way to make it clear that he does not have the power to enter into a contract without their approval o Pension contracts Involves a long period of time, company funds and who knows for how long Whether it is extraordinary or not is a question for the jury o To defeat a claim of apparent authority Make it clear to third party that agent doesnt have authority Argue that it was unreasonable for them to believe he had authority Publicly held corporations Special rules that apply to them that do not apply to closely held corporations Generally federal securities law State corporation law is the same for public and close corporations More federal securities law applies to public than close corporations Two theories Publicly held corporation is purely an organization of private property o Private people invest private money to invest shares of the company and run it as a purely private organization to run it for the profit of the owners (the shareholders) o Purpose of a corporation is to generate profit for the shareholders Concept of incorporation is just a social tool designed to have social ends so people can be employed and businesses can conduct business o Invested in by shareholders who are so numerous as to represent the public The shareholders in public corporations are more distant than those in close corporations o Corporation should be concerned about all stakeholders (everyone in the community who has a stake in the company) rather than just shareholders o Concepts to think about Social responsibility Alternative constituencies Shareholder passivity Institutional investors Market for corporate control Who owns corporation? Most common answer is shareholders but this isnt entirely true o Shareholders dont really have free control to elect directors b/c theyre limited as to who they can elect o Just like citizen doesnt really control country Directors, by statute, have control o Not really the reality Executive officers are usually the ones who are controlling the board Mutual fund Company buys a basket of different stock and then you buy a piece of that company Retirement funds Like mutual funds, the success of it depends on performance of stock Market discipline Management is thinking that if they dont do well theyll be taken over by some outside force that will recognize they havent been managing company right fire them and get new management 46

Jobs depend on making sure stock price doesnt slide too low or else going to be brought out If more people are selling than buying stock, stock price will drop Companies want to buy other companies b/c thats how you make big bucks by buying poorly working companies, fixing them, and selling them for a higher price Increasing stock prices o Reporting good earnings o Management does short term things to keep the price from increasing, and even to increase the prices o Important to be able to have long term thinking to some point so the company can stay in business and keep doing well Sarbanes Oxley Act Congresss attempt to tighten up the reporting rules of publicly held corporations o Sunshine theory and efficient markets o More honest reporting and might not be able to get away with the fraud they had been getting away with Goes back to stock market crash of 1929 o One of main causes was fundamental frauds in market Congress came up with federal securities law as the solution to big companies and the parties within them acting inappropriately o Selling stock not worth what it was supposed to be o Misinforming public and shareholders Securities Act of 1933 Public offering of securities must be registered or have an exemption Any time anyone offers a security to the public must file a registration statement Securities Exchange Act of 1934 Much more broad and multifarious Addresses a lot of problems o Shareholder estrangement problem Shareholders were not connected well enough to company Corporate officers could get away with corporate murder without the shareholders being able to do anything about it Solution improve corporate democracy Proxy process Implications of being public Laws, rules and regulations that apply exclusively through federal law to a publicly held company o Proxy regulation o Tender offer regulation o Periodic reporting o Short swing profits o Sarbanes Oxley Defining a public company 1934 Act 12(a), 12(g), 15(d) pg. 589 Any company that offers it stock to the public (completes a public offering) o Will make you a public company at least during offering if not afterwards What was the effect of that public offering? Are you public enough to be? o Define by the number of shareholders, quantum of capital and whether stock is listed for sale on national exchange 500 shareholders 47

10 million in assets Lots of companies have 10 million dollars in assets, so 500 shareholders is what really separates it To deregister you have to get below 300 shareholders Buy back stock from shareholders Reverse stock split o Were going to issue new stock, if you own 100 or more shares, you will get 1 share of the new stock, if you owe fewer than 100 shares well give you cash Forces people to sell stock

Proxy process State law MBCA 7.22 o Must be one annual meeting of shareholders MBCA 7.01 Can take an action without a meeting as long as all shareholders send a written consent form agreeing that corporation can take that action MBCA 7.04 Its possible to have more than one it would be called a special meeting MBCA 7.02 At meetings you elect directors Need quorum o Look at shares not shareholders Proxy absentee ballot o Allows shareholders to send it in so that they are represented at the meeting o Company must also send out proxy statement and annual report Proxy statement Here are the issues that are going to be presented at the meeting and heres what you need to know about that Inform you about the specific issues at the meeting Annual report Gives you the big picture of the company including financial information so you can put these specific issues in context Ensures that the company tells an accurate, complete and potentially useful story 1934 Act 14(a) and Regulation 14A pg. 589 o Unlawful to solicit a proxy unless you follow these rules if the company has stock registered pursuant to section 12 (does not apply to close corporations) Therefore close companies may solicit proxies If you are a section 12 company you have to follow a bunch of rules before sending out proxies o What if you lie in a proxy statement? Form of securities fraud under Regulation 14A-9 When you publish these materials they must be accurate you cannot omit material facts from them and you cannot supply material misstatements complete and accurate o 14A-8 shareholder proposal Access to the proxy statement for shareholders If the shareholders have a good proposal for the company you can get it into the proxy statement Not allowed to have any proposal that affects an election of the directors

48

If you said these directors could not come from a certain country that would affect election of directors b/c would eliminate some possibilities Producing your own proxy materials Only way for a shareholder to get his own slate of directors in for election Solicit proxies in competition with the company Produce a proxy statement which would only deal with the one issue Would have to comply with all the rules for proxy statements Please vote for my people, heres why, dont vote for those people, heres why Competing proxy statement Expensive to make proxy statement o Need a lawyer How do you find out who the shareholders are to send the proxy statement to? MBCA 18.02 o Ask company to give list of shareholders under 14A-7 for a legitimate purpose MBCA 7.20 o Says you either get the list or you provide the proxy materials in envelopes with stamps to the company and they will send it out This is most likely the option corporation will take b/c list has a lot of personal information and theyre afraid you might misuse it Proxy solicitation The act of talking to other shareholders about the issue is the first step When you speak to other shareholders or send them a letter you solicit a proxy and you did not provide a legitimate proxy statement or follow the other laws or proxy solicitation If you dont provide that information at the very first step the company will get an injunction against you Not necessarily a formal thing but its any time you ask someone for a proxy or take the first step of doing that System is loaded for incumbent management Very little shareholder can do to beat the system of incumbent management without a lot of money and very good lawyer

Tender Offer Bad management depresses stock price and it looks cheap to buy Someone comes in and offers to buy the stock from the shareholders Please tender us your shares and well tender you a good price Regulated by Williams Act 13 and 14 Good for company but makes it more difficult to get rid of management Reporting Annual and quarterly reports Must be filed with SEC and available to public Short Swing Profits Once you are public, your officers, directors and anyone that is a 10% or more shareholder those people may not sell or buy share in their own company without following certain rules o Any time those people buy and sell or sell and buy stock in their own company two transactions within a 6 month period, any profit generated goes to the company Can sell first and buy later or buy first and sell later 49

o Any profit generated goes to the company Disgorgement of profits o Irrebutable presumption is that you had inside information and you used that information to accomplish a short swing transaction How to get around a borrowing limit in the articles Sell bonds (or something else) with a repurchase (repo) option in a certain amount of time Buy them back at the price they were sold for Bank has to promise that they would be willing to sell them back to the corporation in that time If the value of a bond continues to be high, the corporation will buy it back for the amount it was sold for If the value of the bond drops, the corporation probably wont buy it back If everything goes well the bank wont make any money and if everything goes poorly they will lose money Fiduciary duties Directors Responsibilities o Changes to articles o Issuance of new securities (at least authorization of issuance) MBCA 6.21 o Major capital restructuring o Bankruptcy o Major business decisions Do we want to do things that are different from what weve done in the past? o Who will we hire and fire with respect to our officers? Discipline and reward of officers Apply to both public and close corporations Directors have ultimate responsibility and fiduciary responsibilities Two categories of responsibilities o Duty of trust, loyalty, faithfulness Duty of the heart Did the director lie cheat or steal? Thought of someone elses interest ahead of the corporation Took some advantage that shouldnt have been taken o Duty of care Duty of the mind Did the director exercise his or her intellectual capacities appropriately in dealing with this question? Gather information, make a rational judgment o Law is different with respect to these two duties Enforcement o Shareholders can bring a shareholder derivative suit MBCA 7.40 Not bringing it as a personal matter not saying youve been personally injured youre saying the company is being injured by its own directors Standing MBCA 7.41 Must be a shareholder at the time of the injury Must be acting in the interest of the company o Why doesnt the company sue the directors? Directors make a decision about bringing a lawsuit They arent going to sue themselves for doing something wrong 50

Shareholder stands up as the champion of the company and says I will bring an action for the interest of the company its derivative of the companys interest Two views on how process works Demand process MBCA 7.42 If you as a shareholder want to sue the board or you want the company to sue anyone else, you as shareholder need to ask board first please bring an action against these wrongdoers Board will respond by saying no in most circumstances o Board will say they know there is this problem and we dont want to do what you want us to do about it o If its against the board then they might have other reasons not to want to bring it 90 day period board gets to think about it and make a response and if no response within that time then shareholder can file suit o If its refused then the shareholder can still bring action but its just the start Delawares process Shareholders, if they are suing the directors, can file the suit and say we dont have to make a demand b/c it should be excused b/c were suing the people upon whom demand would be made All that seems to do is bring litigation on whether the demand ought to have been excused o Adds layer of litigation potential that you would not have in the above process Steps Youve made your demand Board will most likely Create a special litigation committee o Who is not conflicted in this situation (not accused of wrongdoing) o Pick out board members and say youre a SLC go off and study this demand and the underlying factual assertions and come back with a recommendation o Find facts, determine legal implications of facts (with help of legal counsel), I there any real potential liability in this circumstance, study impact of potential litigation on the business of the company (all facts, not just litigation facts) Who will be involved in litigation? How much will they be involved? Will they be distracted? Will it affect potential relationships? Report will say we recommend this litigation be dismissed or not proceed b/c of these facts o If they find in good faith that it is not in the best interest of the company to proceed with litigation when the suit is filed the company will move to dismiss and the court must dismiss it If you are the litigator and youre representing the shareholder you should attack the good faith of SLC members 51

Show they were conflicted in some way Could be theyve done sloppy work and the facts dont justify the conclusion Conflict o Not just b/c youve been sued o Financial conflict that suggests loyalty issue o Personal conflict (family members of people sued, etc.) o Probable liability If you have a decision making group and the majority of it is disinterested then still not conflicted But if it would turn out that the directors would perceive that they would get stuck with liability (substantial likelihood to be held liable) then they become conflicted in a way that decision not to go forward with suit would be bad faith o Appropriate disinterest and kind of position to recommend not bringing a lawsuit? Standing MBCA 7.41 If you want to bring an action as a shareholder you must have been a shareholder at the time the breach occurred Fairly and adequately represent the company (youre not conflicted in some way) Courts stay the action while the SLC does its work MBCA 7.43 Dismissal MBCA 7.44 Company will move to dismiss case Settlement MBCA 7.45 Must be approved by the court Look at terms of settlement, impact on company and fees and costs Caremark Payment of expenses MBCA 7.46 o May order corporation to pay Ps attorneys fees depending on whether the proceeding resulted in a substantial benefit to the corporation o Court may order P to pay corporations reasonable expenses if suit was created or maintained for an improper purpose or without reasonable cause Potential malpractice case against Ps lawyer o If any of the motions or anything were frivolous then the party initiating them get the bill Duty of care MBCA 8.30 o Is accepting a repo agreement (like above) a violation of duty of care for bank board? Litwin v. Allen Court says irrational decision Could be rational b/c For length of time of repo agreement will be getting interest on bonds Just like a loan almost getting bonds as security would be done in a regular loan o At end of six months can pay off loan and give back price 52

o o

Bank keeps bonds if loan isnt paid back (bank keeps collateral) The board of a bank has a different level of care than a regular board Should not take risks Have to take some risks but you dont want them to take too much risk b/c its supposed to be a secure financial institution If it is an ordinary institution making a somewhat risky decision with some upside potential it wouldnt have been an issue Decision If there is a decision a conscious action (could be a decision to not take action) it will be protected by the business judgment rule Business judgment rule Shlensky v. Wrigley Judges acknowledge theyre not business people Dont want to get into business of second guessing business decisions Going to give them the benefit of the doubt Assumption that the decision is In good faith Has an informed basis In the best interest of the company Does not apply to duty of loyalty To overcome, shareholder will have to prove that Decision harmed the company Decision was not in good faith Decision not on an informed basis Smith v. Van Gorkom o Appropriate time to make decision o Review of the documents o Listening to opposition o Reliance on conflicted person? Bad o Will always be reasonably informed if relied on an expert Brehm v. Eisner Decision not in the best interest of the company Raincoat provisions MBCA 2.02(b)(4) Cannot sue directors unless it is lying, cheating, stealing, kinds of behaviors Put into articles of incorporation Oversight Not making a decision on wheat employees should or should not do Should directors be responsible for employees acts and how responsible should they be? Conscious decision not to follow up on known illegal conduct Results in liability Moves towards the decision end of it Not proper procedures in place (auditing, etc.) so that director did not know about it Duty to have reasonable oversight or monitoring mechanisms in which it is likely that the reports will come up to the board or at least dealt with Must be a process in place that would reveal problems in the company that would be injurious to the company including violations of law problems 53

Utter failure to oversee No legitimate argument for being a director in title only o Duty is to pay attention o Not a defense to not pay attention to what was going on Gross negligence liability Lack of good faith if you dont go to meetings or carry out your responsibilities at all (duty of loyalty issue which overcomes raincoat and BJR) Negligence Maybe didnt take into account some important facts BJR comes to aid of directors Gross negligence No information but still trying to act in best interest of company Lacked some process, information or assimilation or discussion of that information or the time needed to make that decision BJR probably wont offer protection Raincoat provision would offer protection Conscious disregard Takes you into a good faith problem Good faith is a species of loyalty even though applies to both care and loyalty If no personal gain by director, then most likely pretty much a conscious disregard of care duties o Care violation is so egregious that no longer a mental lapse, its a faith lapse BJR wont protect

Waste o Straddles duty of care and duty of loyalty o Giving away assets of the company for nothing in return Duty of loyalty MBCA 8.31 o Conflict of interest Conflict arises when the deal is made There is some sort of a transaction between the company and something thats associated with a director In one sense you could say the directors are sitting on both sides of the table when the negotiation or transaction is made Classic situation is director selling property to the directors own company Three alternatives to remedy a conflict MBCA 8.62 o Disinterested directors vote and you get a quorum o Decision really made by disinterested directors who are protecting companys interest o Allows interested director to negotiate as hard as he wants to MBCA 8.63 o Disinterested shareholders vote and you get a quorum o Not director at issue and not related to director in a way that would cause a problem MBCA 8.61(b)(3) 54

Entire fairness Transaction is fair to the corporation and to all who would have special interest in it o Meaning 8.60(6) Price (fair market value) Was the property needed? Is it a rational transaction for the corporation to undertake? Is it just one of those things that director needs to sell something quick and the company just buys it to help director out For a conflict of interest to turn into a breach of duty, conflicted director must be taking something for himself to the exclusion of others who have an equal right to that Sinclair Oil Corp. v. Levien Ex: not a wholly owned subsidiary, parent company employees on subsidiary board, other subsidiaries break contract with this subsidiary, board members of subsidiary cause no suit to be brought b/c protecting parent company o Business opportunity Not shared with the corporation American Law Institute Approach Pg. 804-805 ALI 5.05(b) (1) fairness test o Looks at behavior of person and is it fair for person to take that opportunity in that way (2) line of business test Sometimes a company will make you sign an agreement saying everything you develop while working at the company is the companys property Should check with the corporation to make sure they dont want the idea or think its theirs before you do anything with it Executive Officers duties Subject to similar duties as directors Shareholder duties Majority shareholders b/c they can exert effective control over the company stand in the same role as a board of directors o Need to exercise power appropriately duty of care and loyalty o If there are any red flags when attempting to sell your shares then the majority shareholder is not allowed to sell their stock to that person Cannot hand over stock to someone who might do harm to the company o If a company has a lot of cash in it and you can get in control of the company you can take action to get that cash in your pocket criminal in many cases Fraud in connection with purchase of securities Lying about securities Rule 10b-5 youre in big trouble if you lie to someone while selling or purchasing a security Applies to both close and public corporations o Every company, every person is potentially a perpetrator or victim of a 10b-5 violation Applies to any security o Stock, bonds, investment contracts, etc. Any fraud in purchase or sale is a violation 55

Its only about fraud o Misstatement o Deceit o Requires intent or recklessness (scienter) Must involve interstate commerce o Very few dont and we wont deal with one Essentially about being a principal violator o Must be involved in the violation itself and presumably have some guilty knowledge and scienter Not many transactions that dont have an implication for corporate stock or bond Basically every decision the board makes or does not make will have some impact potentially on value of the shares Standing You must have either purchased or sold a security Throw out a whole bunch of cases where there was fraud committed in the course of a POTENTIAL sale or purchase of securities Cant say if I would have bought the stock I would have been injured Must be intentional or reckless Negligence is not enough Ernst & Ernst v. Hochfelder Private right of action Not express Infer a private cause of action from the nature of the statute o When Congress make a law or those who promulgate rules legitimately under it there are a bunch of standards about whether you can infer a private cause of action Get your shares back and any damages you suffered How can you commit fraud? Intentional misstatement of fact (lying to someone) o Comes about when a company calculatingly deceive the public about their company while theyre selling securities o Lying to shareholders to be able to purchase the shareholders securities from them Omission of material fact o Insider trading Insiders can be pretty much anyone with possession of material inside information Question is whether they can use that in some way? Buy or sell on the basis of that Materiality depends on accuracy and magnitude of information o Something that is highly accurate but not that big would be material o Something that is huge but we are not sure about its accuracy would be material o Something that is small and were not sure of its accuracy probably isnt material This is not fair and the insiders have a duty to act in a particular way that is above and beyond (fiduciary sort of duty) You have a duty to the shareholders as an insider You have a duty to your company, to the shareholders of your company, not to take advantage of your position for your own gain

56

You have taken information that is not yours, it is your companys and used it to your own advantage

Classic Situation Company decides to cut the dividends, company knows this will make the stock go down in price, a board member runs out to call stockbroker and tell him to sell the stock o Avoid the loss that would come when the announcement was made o You have inside information about the dividends o Its material b/c you are fairly certain it will affect the stock price In an arms length transaction (buyer and seller dont have a relationship of trust), if there is no security involved, you can omit material information that you believe to have about their property Texas Gulf Sulphur Co. If there is a relationship of trust you cannot take advantage of your own superior knowledge Old Law Equal Access Rule Fairness in market requires that buyers and sellers of stock should have equal access to the information If someone had some special information they were violating Rule 10b-5 If you had material inside information that is non-public, you have a duty to disclose the information to the market before you trade or not trade at all o Disclose or abstain o If you dont disclose you will violate the equal access rule o If youre an insider you cannot disclose b/c you would be violating your fiduciary duty to the company to maintain its own secrets Chiarella v. US Person learns from confidential documents that Company A is about to takeover Company B and so he buys a bunch of Company B stock Duty to disclose depends on whether he had a duty to person on other side of transaction o Got his information from being an employee of a company employed by Company A so he indirectly owes a duty to Company A and its shareholders Had no connection to Company B o Had material information and could not have used it to take advantage of anyone in Company A but could take advantage of Company B all he wants o When an insider has secret information and they trade with a shareholder of their own company they breach the fiduciary duty thats owed by trading without revealing the information Misappropriation Theory o Rule 10b-5 says the real heart of the violation is deception or manipulation (lying) and one way of doing that is theft its a deceitful act 57

When you owe a duty to keep the confidence of Company A and you use information you got from Company A to buy stock from Company B you are breaching the duty to keep confidence of Company A and add it to the Company B transaction and this creates a 10b-5 violation o This connection was held to be enough If I tell someone about my information (whether intentionally or unintentionally) they (the tippee) cannot use that information OHagan Lawyer at a firm found out his firm was representing a bidder (Company A) that was about to tender for a target (Company B) and bought a whole bunch of Company B stock o Wasnt working on the transaction but knew about it o Misappropriation charge o Virtually identical to Chiarella However innocently he came about the information he breached the duty to keep it confidential Barry Switzer Heard secret information at a track meet, bought stock for the company Had no duty to anyone involved All 10b-5 violations for insider trading start with the identification of a duty to someone to keep the secret, violation of the duty and a connected transaction (purchase or sale of securities) that happens b/c knew that information Need a connection between the deceit and the purchase or sale of the stock if there is no connection then not a violation Dirks v. SEC Ex-employee calls insurance investment firm and tells him his former company is a fraud o Investment firm has clients sell stock o Tells SEC about fraud Duty came from the person in the company that gave him information Tippees have a duty to abstain when they know the person theyre getting information from has a duty not to pass it along o Knows or should know the information giver (tipper) is violating their duty If dont know or shouldnt know then no duty violation If there is payment for info then you should know (quid pro quo) that there was a duty

Enron Would transfer something to an SPE (Special Purpose Entity) and be paid for it in return and they would put that payment on the books as income o It would be a sham transaction o If they had debt, they would hand it over to an SPE and then it wouldnt be on their books Created SPEs to create transactions that were frauds, to offload debt that was their own and to make their books and performance look better 58

SPEs were actually controlled by insiders who were skimming money off the top of the transactions Lawyer/Accountant liability If a lawyer knows what he is writing was fraudulent then he is a criminal himself o No place to hide by saying I told them they shouldnt do that but I did assist them b/c its their choice CPA v. ROA (regular old accountant) o CPAs obligation is to the public whereas regular accountants is not o At annual shareholder meeting the shareholders approve the CPAs CPA is the shareholder watchdog Taxation C Corporation Corporation will be taxed on all assets and then if corporation distributes dividends to shareholders the shareholders will be taxed individually as well S Corporation Only allowed for small businesses and meet certain other qualifications Taxation as a partnership o Corporation is not taxed, the shareholders are If want S corporation taxation can only have common stock

59

Securities Exchange Act of 1934

PROXIES

14(a). It shall be unlawful for any person, by the use of the mails or by any means or instrumentality of
interstate commerce or of any facility of a national securities exchange or otherwise, in contravention of such rules and regulations as the Commission may prescribe as necessary or appropriate in the public interest or for the protection of investors, to solicit or to permit the use of his name to solicit any proxy or consent or authorization in respect of any security (other than an exempted security) registered pursuant to 12 of this article. REGISTRATION REQUIREMENTS FOR SECURITIES

12(a). It shall be unlawful for any member, broker, or dealer to effect any transaction in any security (other
than an exempted security) on a national securities exchange unless a registration is effective as to such security for such exchange in accordance with the provisions of this title and the rules and regulations thereunder. ***

12(g)(1). Every issuer which is engaged in interstate commerce, or in a business affecting interstate
commerce, or whose securities are traded by use of the mails or any means or instrumentality of interstate commerce shall (A) Within 120 days after the last day of its first fiscal year ended after the effective date of this subsection (7/1/1964) on which the issuer has total assets exceeding $1,000,000 and a class of equity security (other than an exempted security) held of record by 750 or more persons; and (B) Within 120 days after the last day of its first fiscal year ended after 2 years from the effective date of this subsection on which the issuer has total assets exceeding $1,000,000 and a class of equity security (other than an exempted security) held of record by 500 or more but less than 750 persons, Register such security by filing with the commission a registration statement. ***

12(g)(4). Registration of any class of security pursuant to this subsection shall be terminated 90 days, or
such shorter period as the Commission may determine, after the issuer files a certification with the Commission that the number of holders of record of such class of security is reduced to less than 300 persons. ***

12(g)(5). For the purposes for this subsection the term class shall include all securities of an issuer which
are of substantially similar character and the holders of which enjoy substantially similar rights and privileges. ***

60

American Law Institute Principles of Corporate Governance 5.05. Taking of Corporate Opportunities by Directors or Senior Executives
(a) General Rule. A director or senior executive may not take advantage of a corporate opportunity unless (1) The director or senior executive first offers the corporate opportunity to the corporation and makes disclosure concerning the conflict of interest and the corporate opportunity; (2) The corporate opportunity is rejected by the corporation; and (3) Either: (A) The rejection of the opportunity is fair to the corporation; (B) The opportunity is rejected in advance, following such disclosure, by disinterested directors, or, in case of a senior executive who is not a director, by a disinterested superior, in a manner that satisfies the standards of the business judgment rule; or (C) The rejection is authorized in advance or ratified, following such disclosure, by disinterested shareholders, and the rejection is not equivalent of waste of corporate assets. (b) Definition of a Corporate Opportunity. For purposes of this Section, a corporate opportunity means: (1) Any opportunity to engage in a business activity of which a director or senior executive becomes aware, either: (A) In connection with the performance of functions as a director or senior executive, or under circumstances that should reasonably lead the director senior executive to believe that the person offering the opportunity expects it to be offered to the corporation; or (B) Through the use of corporate information or property, if the resulting opportunity is one that the director or senior executive should reasonably be expected to believe would be of interest to the corporation; or (2) Any opportunity to engage in a business activity of which a senior executive becomes aware and knows is closely related to a business in which the corporation is engaged or expects to engage. (c) Burden of Proof. A party who challenges the taking of a corporate opportunity has the burden of proof, except that if such party establishes that the requirements of Subsection (a)(3)(B) or (C) are not met, the director or the senior executive has the burden of proving that the rejection and the taking of the opportunity were fair to the corporation. (d) Ratification of Defective Disclosure. A good faith but defective disclosure of facts concerning the corporate opportunity may be cured if at any time (but no later than a reasonable time after suit is filed challenging the taking of the corporate opportunity) the original rejection of the corporate opportunity is ratified, following the required disclosure, by the board, the shareholders or the corporate decisionmaker who initially approved the rejection of the corporate opportunity, or such decisionmakers successor. (e) Special Rule Concerning Delayed Offering of Corporate Opportunities. Relief based solely on failure to first offer an opportunity to the corporation under Subsection (a)(1) is not available if: (1) such failure resulted from a good faith belief that the business activity did not constitute a corporate opportunity, and (2) not later than a reasonable time after suit is filed challenging the taking of the corporate opportunity, the corporate opportunity is to the extent possible offered to the corporation and rejected in a manner that satisfies the standards of Subsection (a).
61

Securities Exchange Act of 1934 10. It shall be unlawful for any person, directly or indirectly, by the use of any means or instrumentality of
interstate commerce or of the mails, or of any facility of any national securities exchange *** (b) To use or employ, in connection with the purchase or sale of any security registered on a national securities exchange or any security not so registered, any manipulative or deceptive device or contrivance in contravention of such rules and regulations as the Commission may prescribe as necessary or appropriate in the public interest or for the protection of investors.

Rule 10b-5: Employment of Manipulative and Deceptive Devices


It shall be unlawful for any person, directly or indirectly, by the use of any means or instrumentality of interstate commerce, or of the mails or of the facility of any national securities exchange, (a) To employ any device, scheme, or artifice to defraud, (b) To make any untrue statement of a material fact or to omit to state a material fact necessary in order to make the statements made, in the light of the circumstances under which they were made, not misleading, or (c) To engage in any act, practice or course of business which operates or would operate as a fraud or deceit upon any person, in connection with the purchase or sale of any security.

62

Uniform Partnership Act 2 Definition of Terms


Court, business, person, bankrupt, conveyance, real property

6 Partnership Defined
Association of two or more people, co-owners, business for profit

7 Rules for Determining the Existence of a Partnership


Agreement to receive profits of a business is prima facie evidence of partnership unless received in payment for something

9 Partner Agent of Partnership as to Partnership Business


Partner is agent of partnership for carrying on ordinary business of partnership unless partner is explicitly given no authority and a third party knows that Act of partner for extraordinary business is not binding List of things that all of the partners have to agree on

13 Partnership Bound by Partners Wrongful Act


Partners wrongful act in the ordinary course of business is chargeable to partnership

14 Partnership Bound by Partners Breach of Tort


Partnership is bound where partner acting in the scope of his apparent authority takes money of third party and misapplies it, and where partnership receives money of a third party and a partner misapplies it while it is in the custody of partnership

16 Partner by Estoppel
Person represents himself (or someone else represents him) as partner in partnership he is liable to anyone who relied on that representation, if made publicly then liable to everyone Partnership liability liable as if partner Can bind partners that represented him as a partner because he is an agent of all partners making rep

17 Liability of Incoming Partner


Incoming partner liable for all obligations arising before admission to partnership as if partner when liability incurred (liability assigned only out of partnership property)

18 Rules Determining Rights and Duties of Partners


SUBJECT TO AGREEMENT Share equally in profits, share in losses same as profits Partnership indemnifies partner for liabilities arising during ordinary course of business Any partner payment made over amount of agreed contribution will accumulate interest Receive interest on contribution after date when repayment should be made Equal rights in management and conduct of partnership business Partners shall not be paid a salary for conducting business To add an additional partner, all partners must consent Difference arising as to ordinary course of business matters decided by majority of partners If going to break an agreement between partners must be done with consent of all partners

20 Duty of Partners to Render Information


Must give all true information when requested from another partner

21 Partner Accountable as Fiduciary


Must account for benefit derived without consent of other partners from transaction connected with partnership or by use of partnership property
63

22 Right to an Account
Right to form account if: Wrongfully excluded from partnership business Right exists under terms of agreement Under 21 Other circumstances render it just and reasonable

25 Nature of a Partners Right in Specific Partnership Property


Partner is co-owner of specific partnership property as tenant in partnership Has right to possess property for partnership purposes but needs consent of all partners for other purposes Partners right not assignable unless all partners assign their rights When partnership property attached for debt partner cannot claim any right to property Upon death, right vests in other partners unless last surviving partner then vests in legal rep

26 Nature of Partners interest in the Partnership


Partners interest in partnership and partnership property is his share of profits

29 Dissolution Defined
A partner ceases to be associated in carrying on of partnership business

30 Partnership Not Terminated by Dissolution


Partnership continues until winding up of partnership affairs completed

31 Causes of Dissolution
No violation of agreement between partners Termination of definite term or particular undertaking specified in agreement Express will of any partner when no term or undertaking specified Express will of all partners after termination of term or undertaking Expulsion of partner Violation of agreement between partners by express will of any partner at any time Unlawful to continue with partnership business Death of partner Bankruptcy of partner or partnership Decree of court under 32

32 Dissolution by Decree of Court


Court will decree dissolution when Partner is of unsound mind Partner becomes incapable of performing his part Partner is guilty of conduct that prejudices the partnership Partner willfully or persistently breaches partnership agreement Business of partnership can only be carried on at loss Other equitable reasons

35(1)(b) Power of Partner to Bind Partnership to Third Person after Dissolution


Can be bound by any transaction that would bind the partnership prior to dissolution provided third party extended credit to the partnership prior to dissolution and had no knowledge of dissolution or had not extended credit but knew of partnership and didnt know of dissolution and dissolution was not advertised in newspaper

64

36 Effect of Dissolution of Partners Existing Liability


Dissolution of partnership does not dissolve partners liability Partner discharged from liability through agreement between partner and creditor and person continuing business; can be inferred from course of dealing If partner agrees to assume liabilities of dissolved partnership Individual property of a deceased partner will be liable for all obligations partnership incurred

38 Rights of Partners to Application of Partnership Property


Can apply partnership property to discharge liabilities to partners and the rest will be in cash unless dissolution is caused by the expulsion of a partner then will all be in cash Dissolution caused by contravention of partnership agreement Partners that did not cause dissolution will get above rights and right to damages for breach of contract against partner(s) that breached agreement If the non-wrongful partners want to continue the business in the same name as long as they pay the wrongful partner his interest in the partnership Partner who caused dissolution wrongfully as long as it is not continued will get the top amount minus contract breach damages but if the business is continued will get his interest minus any damages caused by dissolution and released from all partnership liabilities

40 Rules for Distribution upon Dissolution


Rules for settling of accounts among partners Assets of partnership are: partnership property, contributions from partners necessary to pay liabilities Ranking of liabilities Creditors other than partners Partners other than for capital and profits Partners for capital Partners for profits If some but not all partners are insolvent or refuse to contribute, the rest shall contribute their share of liabilities and their share of the remaining liability (from the insolvent partner) A deceased partners individual property is liable for contribution to paying partnership debts

41 Liability of Persons Continuing the Business in Certain Cases


When a new partner joins an existing partnership or a partner assigns his interests and business is continued without liquidation, creditors of original partnership are creditors of the continuing business

42 Rights of Retiring or Estate of Deceased Partner When the Business is Continued


SUBJECT TO AGREEMENT Partner retires or dies, business continued, with no settling of accounts between him and continuing partnership, he may have the value of his interest at the date of dissolution, will be considered an ordinary creditor for that value plus interest or receive the profits according to his right in the property of the dissolved partnership

65

Revised Uniform Partnership Act 306 Partners Liability


All partners are jointly and severally liable for all partnership obligations New incoming partner is not liable for any partnership obligations prior to his admission In a LLP, there is limited liability

1001 Statement of Qualification


How to become a LLP Must be approved by vote necessary to amend partnership agreement Following approval, file statement of qualification which includes Name of partnership; Address of chief exec office; If no office in this state, name and address for agent of service; Statement that partnership elects to be LLP; Deferred effective date, if any Agent of service has to be resident of this state LLP formed on date of filing or if deferred effective date then that, amendment effective when filed Status as LLP and liability of partners not affected by errors or changes in info on SOQ

1002 Name
Must end with Registered Limited Liability Partnership, Limited Liability Partnership, R.L.L.P, L.L.P., RLLP or LLP

1003 Annual Report


Must file annual report that contains all of the above names and addresses Must be filed between Jan 1 and Apr 1 Sec of State may revoke LLP status if fail to file report or pay fee, notice from Sec of State req listed Revocation does not dissolve partnership May reapply within 2 years of revocation if reason has been corrected or did not exist Reinstatement relates back to date of revocation like revocation never occurred

66

Uniform Limited Liability Company Act 101 Definitions in this Act


Articles of organization; at-will company; business; debtor in bankruptcy; distribution; distributional interest; entity; foreign limited liability company; limited liability company; manager; managermanaged company; member-managed company; operating agreement; person; principal office; record; sign; state; term company; transfer

103 Effect of Operating Agreement; Non-waivable Provisions


All members can enter into OA (oral or written) to regulate affairs of company and conduct business, govern relations among members, managers and company OA may not Unreasonably restrict right to info Eliminate duty of loyalty but may identify types of conduct that do not violate duty of loyalty and specify number of members or disinterested managers that may authorize a specific act that would violate duty of loyalty Unreasonably reduce duty of care Eliminate obligation of good faith and fair dealing but may determine standards under which performance is to be measured Vary the right to expel a member Vary the requirement to wind up business Restrict rights of person, other than manager or member

105 Name
Must contain limited liability company, limited company, LLC, LC, Ltd Co Must be distinguishable upon the records of Secretary of State from Name of Corp, LP or LLC, organized or authorized for business in this State Name reserved or registered Fictitious name approved May apply to Sec of St for authorization to use non-distinguishable name, granted if: Present owner of reserve name consents or applicant delivers certified copy of final judgment saying name can be used from a US court May use the name of another company if: Have merged with other company, been formed by reorganization with other company or acquired substantially all assets, including name, of other company

106 Reserved Name


May reserve exclusive use of name by application for filing state name to be reserved, if available will be reserved for exclusive use for nonrenewable 120 day period May transfer the reservation to another person with signed notice to Sec of St

107 Registered Name


Foreign LLC may register its name if distinguishable in new state Must complete a renewal application every year between Oct 1 and Dec 31

112 Nature of Business and Powers


Organized for any lawful purpose Unless articles of org say otherwise, same powers as individual to carry on business list

202 Organization
Organized by one or more persons by filing Articles of Organization Existence begins when filed
67

203 Articles of Organization


Must include: name of company; address of office, name/add of agent; name/add of all organizers; term company?; manager-managed?; member liability? May include: provisions for Operating Agreement; other matters ok by law If OA is inconsistent with AO OA controls as to managers and members and AO controls as to persons who reasonably rely on AO to detriment

301 Agency of Members and Managers


Each member is agent for ordinary course of business unless explicit no authority and third party knew Extraordinary business actions binding only if authorized by all members In manager-managed member is not agent, manager is an agent for ordinary business, extraordinary business only binding if authorized under 404 Every member or manager (depending on management style) may transfer LLC interest in real property, conclusive in favor of third party who had no knowledge of lack of authority

303 Liability of Members and Managers


No personal liability Failure to observe company formalities does not permit piercing the veil If AO says personal liability for all or some then personal liability and if member has consented in writing to adoption of that provision in AO

404 Management of LLC


In member-managed each member has equal rights in management, any matter may be decided by majority of members (except for some) In manager-managed each manager has equal rights in management, any matter decided by majority of managers (if only one then him), manager must be elected by majority of members and hold office until successor elected unless manager resigns or removed Matters requiring consent of all members Amendment of OA Authorization of acts that would violated duty of loyalty Amendment to AO Making of interim distributions Admission of new member Consent to dissolve company Waiver of right to have companys business wound up and business terminated Consent to merge with another company Sale of all or substantially all companys property No need to have meeting to get consent of members or managers Can use proxy voting

405 Sharing of and Right to Distributions


Any dist. made before dissolution and winding up must be in equal shares Member has no right to receive a dist. If member becomes entitled to receive dist. member has status and rights of creditor

68

Model Business Corporation Act 2.01 Incorporators


One or more persons that files Articles of Incorporation

2.02 Articles of Incorporation


Must include: name, number of authorized shares, address of office, address of incorporators May include: addresses of initial directors (should put in bylaws instead), purpose, regulating affairs, regulating powers of corp., directors and shareholders, par value for shares, any personal liability, raincoat provision, indemnification provision

2.03 Incorporation
Formation upon filing of AI

2.04 Liability for Preincorporation Transactions


Persons saying they are acting on behalf of the corp when they know there is no corp are jointly and severally liable

2.05 Organization of Corporation


After filing if initial directors name in AI, will hold organizational meeting to appoint officers and adopt bylaws If initial directors not named in AI, incorporators will hold org meeting to elect directors and complete org or elect directors who will complete org No meeting needed if there is written consent describing action taken and signed by incorporators

4.01 Corporate Name


Must contain corporation, incorporated, company, limited or corp, inc, co, ltd If purpose stated in AI cannot imply a different purpose Must be distinguishable upon the records from any corporate name or fictitious name May apply to use non-distinguishable name (for details look at ULLCA 105 same thing)

6.01 Authorized Shares


AI must state any classes of shares and number of shares of each class authorized to issue for special classes the AI must describe the terms of the shares (preferences, rights, limitations) all shares of a class must be identical AI must authorize one or more classes that have: unlimited voting rights and liquidation rights AI may authorize on or more classes with: special voting rights, redeemable or convertible, distributions calculated in any manner (ex: cumulative v. non-cumulative), or distribution preferences

6.03 Issued and Outstanding Shares


May issue # authorized, shares issued are outstanding until reacquired, redeemed, converted or canceled Need to at least have one or more outstanding shares that together have unlimited voting rights and liquidation rights

6.20 Subscription for Shares Before Incorporation


Irrevocable for 6 months unless agreement provides longer or shorter Directors determine payment terms unless agreement specifies them Fully paid and nonassessable when corp receives consideration specified in agreement If default in payment corp may collect amount as if any other debt or rescind agreement and sell shares if debt remains unpaid for more than 20 days after corp sends written demand (unless otherwise agreed)

69

6.21 Issuance of Shares


Powers in this granted to directors may be reserved to shareholders by AI Directors may authorize shares issued for consideration (property, cash, promissory notes, services, contracts for services or other securities) Directors must determine that consideration is adequate before issuance If adequate and consideration provided then shares are fully paid Can place shares in escrow until future benefit received Issuance of shares requires approval of quorum of shareholders if: Shares issued for consideration other than cash or cash equivalents Voting power will comprise more than 20% of voting power of shares of corporation that were outstanding immediately before transaction

6.22 Liability of Shareholders


Shareholder must pay consideration for which shares were authorized to be issued No personal liability for debts or act of corporation, can become personally liable by own conduct

6.25 Form and Content of Certificates


Shares may but dont have to have a certificate rights and obligations stay the same Each share certificate must state: name of corp, state of incorp, name of shareholder, and number and class of shares certificate represents If corp authorized to issue different classes of shares rights for each class and variations in rights must be summarized on certificate or must say will furnish shareholder info on request in writing Two officers (designated in bylaws or by directors even if officers dont hold office at time of issuance) must sign certificate and may bear corp seal

6.26 Shares Without Certificates


Unless AI or bylaws provide otherwise, can issue shares without certificates Must send written statement of info required on certificates within reasonable time after issuance

6.30 Shareholders Preemptive Rights


Shareholders do not have preemptive rights unless AI provides so Statement included in AI the corporation elects to have preemptive rights means: shareholders have right to acquire proportional amounts of corps unissued shares when board decides to issue them Shareholder may waive preemptive rights in writing (irrevocable even if consideration provided) No preemptive right for: shares issued as compensation to directors, officers or employees, shares authorized by AI that are issued within 6 months from effective date of incorp, shares sold for something other than money No preemptive rights if your shares dont have voting rights but have preferential distribution rights No preemptive rights for preferential distribution shares unless convertible into non-preferential if your shares have voting rights but dont have preferential distribution rights If preemptive rights are not exercised, those shares may be issued to any person for 1 year after being offered to shareholders consideration cannot be lower than that set for preemptive rights After 1 year can be offered at lower consideration but must go back through preemptive rights again

6.31 Corporations Acquisition of Its Own Shares


Corporation may acquire own shares become authorized but unissued shares If AI prohibits reissue of acquired shares, # of authorized shares reduced by # of acquired shares

70

6.40 Distributions to Shareholders


Directors may authorize distributions subject to AI and below Record date (if not stated) is date directors authorized distribution No distribution may be made if: corporation would not be able to pay debts as they come due or assets would be less than sum of liabilities plus amount needed if corp were to be dissolved at time of distribution Directors can determine this through financial statements or on fair valuation or other method reasonable in circumstances Indebtedness to shareholder b/c of distribution is equal to corps indebtedness in general

7.01 Annual Meeting


Unless directors elected by written consent (7.04), corp shall hold annual meeting of shareholders at place and time fixed in bylaws, but if AI authorizes cumulative voting, directors must be elected by unanimous written consent If no place stated in bylaws then held at principal office Failure to hold annual meeting does not affect validity of corporate action

7.02 Special Meeting


Special meeting of shareholders directors call for it or if 10% of the votes (unless otherwise provided no more than 25%) to be cast on an issue deliver a written demand for a meeting describing purpose of meeting demand may be revoked Record date is date first shareholder signs demand if not otherwise provided Held at place of bylaws or in principal office if no place in bylaws Only business within purpose of meeting notice may be conducted at special meeting

7.04 Action Without Meeting


No meeting required if action taken by all shareholders entitled to vote on action, action must be evidenced by written consents signed by all shareholders entitled to vote filed with corp records AI may provide that action required to be taken without prior notice if consent in writing are signed by minimum # of votes required to authorize action Different record dates listed Consent form = a vote If notice need to be given to nonvoting shares requirements Can use an electronic transmission to consent if can be determined that authorized by shareholder

7.20 Shareholders List for Meeting


After fixing record date for meeting, corp will prepare list of all shareholders entitled to notice arranged by voting group and show address and # of shares held by each Must be available for inspection by any shareholder at principal office, shareholder can demand inspection and copy at his expense List available at meeting If corp refuses, court may order inspection and copying at corps expense and postpone meeting until complete

71

7.22 Proxies
Shareholder may vote his shares in person or by proxy Appoint proxy to vote by signing appointment form or authorized electronic transmission Appointment effective when form received by person authorized to tabulate votes valid 11 months unless longer period expressly provided in form Revocable unless form states it is not revocable and couple with interest appointment of pledge, person who purchased shares, creditor of corporation, or party to voting agreement Death of shareholder appointing does not affect right of corp to accept proxys authority unless notice of death is received by person authorized to tabulate votes before proxy exercises authority

7.25 Quorum and Voting Requirements for Voting Groups


Quorum = majority of votes entitled to be cast on matter, cannot take action if quorum doesnt exist When share is present for any purposes at meeting it is present for all purposes Within quorum must have majority votes to take action voted on Can change this in AI

7.28 Voting for Directors; Cumulative Voting


Unless otherwise provided, directors elected by plurality of votes cast by shares when quorum present No right to cumulative voting unless AI specifies all shareholders are entitled to cumulate their votes for directors multiply number of votes entitled to cast by number of directors up for election and distribute votes among one or more candidates If entitled to vote cumulatively cannot be unless meeting notice states cumulative voting authorized or shareholder gives notice to corp more than 48 hours before meeting of intent to cumulate gives all other shareholders right to vote cumulatively

7.30 Voting Trusts


Conferring trustee right to vote and act for them sign agreement with provisions of trust and transfer shares to trustee trustee must prepare list of names of owners in trust and # and class of shares and deliver copies of list and agreement to corps principal office Effective date is date first shares registered in trustees name valid for only 10 years unless extended All or some of members can extend trust past 10 years by written consent lasts another 10 years

7.31 Voting Agreements


Sign agreement for manner in which parties will vote their shares specifically enforceable

72

7.32 Shareholder Agreements


Can eliminate board of directors or restrict discretion or powers of board Can govern the authorization of distributions subject to 6.40 Can establish who will be directors or officers and their terms in office, manner of selection or removal Can govern exercise of voting power incl. use of weighted voting rights Can establish terms of any agreement for transfer or use of property or services between corp and shareholder, director, officer or employee or among any of them Can transfer to one or more shareholders authority to exercise corporate powers or manage business (incl. resolution of deadlock among directors or shareholders) Requires dissolution of corp. at request of shareholders or upon occurrence of an event Can exercise corporate powers and management and relations between shareholders, corp, directors Agreement must be set forth in AI or by laws or approved by all shareholders at time of agreement, subject to amendment (unless otherwise provided) and valid for 10 years (unless otherwise provided) Agreement must be noted on share certificates if outstanding shares, must be recalled to put on If no notice then purchaser entitled to rescind purchase within 90 yrs after discovery of agreement or 2 years after purchase Agreement ceases to be effective if become public corp If no directors, fiduciary duties are placed on persons with what would be directors discretion No piercing of the veil because of following the agreement even if dispenses with corp formalities

7.40 Subchapter Definitions


Derivative proceeding civil suit on behalf of corp

7.41 Standing
Shareholder may not commence derivative suit unless shareholder: (1) Was shareholder at time of act or omission or shares transferred from someone who was shareholder at time and (2) Fairly and adequately represents interests of corp

7.42 Demand
No shareholder may commence derivative suit until: (1) Written demand made and (2) 90 days expired from date demand made unless rejected by corp before that or irreparable damage would occur in 90 days

7.43 Stay of Proceedings


If corp commences inquiry into allegations, court may stay proceeding for appropriate time

73

7.44 Dismissal
(a) Derivative suit dismissed by court on motion by corp if panel or group in (b) determined in good faith, after reasonable inquiry that maintenance of suit not in best interest of corp (b) Unless panel created, determination made by: (1) majority vote of qualified directors if they are a quorum or (2) majority vote of committee of qualified directors appointed by majority vote of qualified directors (dont have to constitute quorum) (c) if suit commenced after rejection of demand complaint must allege particular facts that either (1) majority of board did not consist of qualified directors or (2) requirements of (a) not met (d) if majority of board of qualified directors at time of determination, P shall have burden of prove (a) not met; if not maj of qualified directors then corp has burden to show (a) met (e) make motion to court to have court appointed panel make determination whether suit is in best interest of corp P shall have burden to prove (a) not met

7.45 Discontinuance or Settlement


Settlement or discontinuance must be approved by court, if will substantially effect interest of corps shareholders then court will require notice to shareholders

7.46 Payment of Expenses


On termination of suit court may: (1) order corp to pay Ps reasonable expenses if suit resulted in substantial benefit to corp (2) order P to pay Ds reasonable expenses if suit commenced or maintained w/o reasonable cause or for improper purpose or (3) order party to pay opposing partys reasonable expenses incurred in making motion if motion was not grounded in fact or warranted by existing law or to harass or cause delay

8.01 Requirements for and Functions of Board of Directors


(a) except under 7.32, must have board of directors (b) all corporate powers exercised under authority of board, business managed by directors, subject to limits in AI or shareholder agreement (c) oversight responsibilities business and performance plans, major risk exposure, performance and compensation of senior officers, policies and practices for compliance with law and ethics, preparation of financial statements, effectiveness of internal controls, arrangements to provide adequate and timely information to directors, and composition of board and committees

8.04 Election of Directors by Certain Classes of Shares


If AI allows classes of shares, AI may also allow election of all or certain directors by holders of certain classes of shares considered a separate voting group for purposes of election of directors

8.05 Terms of Directors Generally


Terms of initial directors of corp expire at first shareholders meeting for election of directors Terms of other directors expire at the next meeting they are up for election Decrease in number of directors does not shorten directors term Term of director elected to fill vacancy expires at next election meeting Unless specified under AI, director continues to serve until directors successor is elected and qualifies or there is a decrease in number of directors
74

8.06 Staggered Terms for Directors


AI may provide for staggering terms of directors by dividing total number of directors into 2 or 3 groups (must be even number of directors in each group) Depending on number of groups, each director will serve for 2 or 3 years

8.20 Meetings
Board of directors may hold regular or special meetings Unless AI or bylaws say otherwise, board may permit meeting to be conducted by any way of communication where all directors may simultaneously hear each other during meeting

8.21 Action Without Meeting


Except if AI or bylaws require action by directors be taken at meeting, action can be taken without a meeting if each director signs a consent describing action and delivers it to corp Action taken under section is act of directors when all consents of all directors arrive at corp Consent may specify an effective date consent may be withdrawn by written revocation delivered to corp as long as not all consents have been delivered to corp

8.24 Quorum and Voting


Unless AI or bylaws require greater #, quorum of directors consists of: Majority of fixed # of directors if fixed board size or majority of directors prescribed if corp has variable range sized board AI or bylaws may authorize quorum of directors to consist of no fewer than 1/3 of directors If quorum present when vote taken, yes vote by majority is act of board unless AI or bylaws require more Director present at meeting when corporate action taken is deemed to assent to action unless: He objects at beginning of meeting to holding meeting His dissent is entered into minutes of meeting or Delivers written notice of dissent to presiding officer

8.25 Committees
Unless AI or bylaws provide otherwise, board of directors may create committees and appoint directors to serve on committees Creation of committee and appointment of members must be approved by greater of (1) majority of all directors or (2) number of directors required by AI or bylaws to take action under 8.24 8.20 8.24 apply to committees as well To extent specified in AI or bylaws or by board, committees may exercise powers of board Committee may not: Authorize distributions unless there is a formula or method or if prescribed by board Approve or propose shareholder action Fill vacancies on board or Adopt, amend or repeal bylaws May appoint alternate members to take over for an absent member but must be by unanimous vote unless specified in AI, bylaws or resolution creating committee

75

8.30 Standards of Conduct for Directors


Directors must act in good faith and in best interest of corporation When becoming informed for decision making or devoting attention to oversight must act as a reasonable person in directors position would act Must tell rest of board any material information to help in making decision, unless would violate a duty imposed by law or professional ethics rules May rely on authority or duty of group in (f)(1) or (3) to perform boards functions unless know of a reason not to rely May rely on information or reports prepared by anyone listed in (f) Director can rely on: Officers or employees of corp that director reasonably believes to be reliable and competent Legal counsel, public accountants or other experts on their matters of expertise Committee of board of which director is not a member if committee merits confidence

8.31 Standards of Liability for Directors


Director not liable to corp or shareholders for decision to or not to take action unless party establishes that: No defense in AI, not protected under 8.61 or protection under 8.70 precludes liability and Challenged conduct was result of Breach of good faith or Decision That director did not believe was in best interest of corp or that director was not informed to extent reasonably appropriate in circumstances or Lack of objectivity due to familial, financial or business relationship or lack of independence due to control by another person have a material interest in conduct That relationship or control could reasonable be expected to affect judgment in a manner adverse to corp and Director has not established that conduct was reasonably believed to be in the best interest of the corp or Sustained failure to devote attention to oversight of business or timely attention by making inquiry when facts of significant concern materialize that would alert reasonably attentive director to need for or Receipt of financial benefit to which director was not entitled or breach of duties to deal fairly with corp and shareholders Party seeking to hold director liable For money damages has to establish that Harm to corp or shareholders suffered and Harm suffered was proximately caused by directors conduct or For other money payment shall have to show payment sought is appropriate in circumstances

76

8.33 Directors Liability for Unlawful Distributions


Director who votes for distribution in access of what may be authorized is personally liable for amount of distribution excess Director held liable for unlawful distribution entitled to: Contribution from other directors who voted for distribution Recoupment form shareholder of pro-rata portion of amount shareholder accepted, knowing distribution was made unlawfully A proceeding to enforce: Liability of director barred unless commenced within 2 years after date on which effect of distribution measure or violation of 6.40(a) occurred or one which distribution was made or Contribution or recoupment barred unless commenced within one year after liability of claimant has been finally adjudicated

8.40 Officers
Corporation has officers described in bylaws or appointed by board Board may elect individuals to fill offices, officer may appoint officers if authorized by bylaws or board Bylaws or board will assign one of officers responsibilities for preparing minutes of meetings and maintaining and authenticating records of corp Same person may simultaneously hold more than one office

8.41 Functions of Officers


Each officer has authority and shall perform functions set forth in bylaws or function prescribed by board or by direction of officer authorized by board to prescribe functions of other officers

8.60 Subchapter Definitions


Defines: Directors conflicting interest transaction, control, relevant time, material financial interest, related person, fair to the corporation, required disclosure

8.61 Judicial Action


Transaction may not be subject of equitable relief or give rise to damages against directors in derivative suit on ground that director has interest respecting transaction if it is not a directors conflicting interest transaction Conflicting interest transaction may not be subject of equitable relief on ground that director has interest respecting transaction, if: Followed 8.62 Followed 8.63 Transaction, judged by circumstances at relevant time, established to be fair to corp

77

8.62 Directors Action


Transaction must be authorized by vote of majority of qualified directors (at least 2) who voted on transaction after required disclosure by conflicted director to qualified directors of information not known by qualified directors provided that: Qualified directors deliberated and voted outside presence of and without participation by any other director and If committee took action, all committee members must be qualified directors and committee composed of all qualified directors on board or members of committee appointed by majority of qualified directors on board If conflicting interest comes from related person (v) or (vi) has a material financial interest in transaction, conflicted director not obligated to make required disclosure to extent director believes doing so would violate duty imposed under law, obligation of confidentiality or professional ethics provided that conflicted director discloses: All information required to be disclosed that is not so violative, Existence and nature of directors conflicting interest and Nature of conflicted directors duty not to disclose confidential information Majority of all qualified directors constitutes a quorum If AI or bylaws requires more for a quorum in this situation, then not qualified directors may participate

8.63 Shareholders Action


Transaction is ok if majority of qualified shares are in favor of transaction after (1) notice to shareholders describing action, (2) telling corporation who is not qualified, and (3) communication to qualified shareholders of pertinent information Director must tell corporation who is not qualified as far as he knows Majority of votes of qualified shares constitutes a quorum If action is taken without informing corporation of who is not qualified to vote, if it can be shown that it did not affect the outcome the court can give effect to the vote

14.07 Other Claims Against Dissolved Corporations (Successor Liability)


Dissolve corporation may public notice of dissolution and request that person with claims present them Notice must: Be published once in a newspaper of county of principal office Describe info that must be included in claim and mailing address to send claim to State that claim will be barred unless suit commenced within 3 years after publication of notice Claimant who was not given written notice Claimant whose claim was timely sent but not acted upon Claimant whose claim is contingent or based on event occurring after effective date of dissolution Claim not barred may be enforced Against dissolved corporation to extent of undistributed assets or If assets distributed in liquidation can bring claim against shareholder but his total liability may not exceed total amount of assets distributed to shareholder

78

14.30 Grounds for Judicial Dissolution


Court may dissolve a corp In proceeding by attorney general if established that Corp obtained articles through fraud or Corp has continued to exceed or abuse authority In proceeding by shareholder (if not public corp) that establishes Directors are deadlocked in management of corporate affairs, shareholders unable to break deadlock and irreparable injury is threatened or suffered or business can no longer be conducted to advantage of shareholders Directors have acted illegally, oppressively or fraudulently Shareholders are deadlocked and have failed for 2 consecutive annual meetings to elect successors to directors whose terms have expired or Corporate assets are being misapplied or wasted In proceeding by creditor if established that Creditors claim has been reduced to judgment, judgment returned unsatisfied and corp is insolvent or Corp has admitted in writing claim is due and corp is insolvent or In proceeding by corp to have voluntary dissolution continued under court supervision In proceeding by shareholder if corp has abandoned business and failed within reasonable time to liquidate and distribute assets and dissolve

14.34 Election to Purchase in Lieu of Dissolution


If not a public corp, corp or other shareholders can elect to purchase shares of shareholder petitioning for judicial dissolution irrevocable Filed with court any time within 90 days after tiling of petition, corp must given written notice to other shareholders in 10 days giving right to others to join in purchase Within 60 days reach agreement as to fair value and terms of purchase, court shall enter order directing purchase If unable to reach agreement court will determine fair value Court will determine the terms and conditions of purchase Court will dismiss petition to dissolve and if petitioning shareholder had probable grounds for relief will order reasonable fees be paid to him

15.05 Effect of Certificate of Authority (Internal Affairs Doctrine)


Certificate of authority authorizes foreign corp to transact business in this state subject to right of state to revoke certificate Same rights and duties as corps incorporated in this state Does not authorize state to regulate organization or internal affairs of foreign corp authorized to transact business in this state law does not apply to that corp

79

Das könnte Ihnen auch gefallen